SlideShare una empresa de Scribd logo
1 de 52
Descargar para leer sin conexión
Danny Perich C.
www.sectormatematica.cl
PSU MATEMATICA:
GEOMETRÍA
Creación y recopilación de ejercicios de geometría, elaborado con el objetivo de
ayudar a los estudiantes a preparar de manera óptima la Prueba de Selección
Universitaria (PSU) de matemática. Este trabajo no persigue fines de lucro, por
lo consiguiente, este texto se distribuye en forma gratuita. ¡¡Éxito‼
2017
DANNY PERICH CAMPANA
www.sectormatematica.cl
Danny Perich C.
www.sectormatematica.cl
Danny Perich C.
www.sectormatematica.cl
1. Las coordenadas del punto simétrico de P(-1, 4) respecto al eje de las abscisas es
A) (1, -4) B) (-1, -4) C) (1, 4) D) (4, -1) E) (4, 1)
2. La(s) transformación(es) que permite(n) obtener la figura 2, a partir de la figura 1 es
A) Rotación B) Traslación C) Simetría Central
D) Simetría Axial E) Traslación y rotación
3. Al trasladar el punto (-4, 2), se obtiene el punto (0, 0), entonces el vector de traslación es
A) (4, -2) B) (4, 2) C) (-4, -2) D) (2, -4) E) (-2, 4)
4. Se forma un segmento con las coordenadas P(1,3) y Q (-5,2). Al aplicársele una traslación, el
nuevo segmento tiene como coordenada imagen de P a P’(-2, 1), entonces la coordenada
imagen de Q es
A) (-3, -2) B) (-8, 0) C) (-4, -1) D) (2, -5) E) (5, -2)
5. ¿Cuál(es) de las siguientes figuras tiene(n) eje de simetría?
I. II. III.
A) Sólo I B) Sólo II C) Sólo I y II D) I, II y III E) Ninguna
6. Se efectúa una rotación de 90º al punto (-5, 4), en torno al origen. Las nuevas coordenadas de
este punto son
A) (-5, -4) B) (5, -4) C) (-4, -5) D) (5, 4) E) (-4, 5)
7. ¿Cuántos ejes de simetría tiene la figura siguiente?
A) 1
B) 2
C) 3
D) 4
E) 5
8. Al trasladar el punto A(5, -3) según el vector (-5, 3), la nueva coordenada de A es
A) (0, 0) B) (0, 6) C) (0, -6) D) (10, 0) E) (-10, 0)
Figura 1 Figura 2
Danny Perich C.
www.sectormatematica.cl
9. El triángulo ABC de coordenadas A(4, 1), B(2, 5) y C(1, 3) se traslada formando el triángulo de
coordenadas A’(7, 3), B’(5, 7) y C’(4, 5). El vector correspondiente a esta traslación es
A) (-3, -2) B) (-3, 2) C) (3, -2) D) (3, 2) E) (2, 3)
10. Al punto de coordenada (2, 0) se le aplica una rotación de 90º en torno al punto (2, 2).
¿Cuál es la nueva coordenada de este punto?
A) (-2, 0) B) (0, 2) C) (0, -2) D) (2, 2) E) (-2, -2)
11. ¿Cuál(es) de las siguientes afirmaciones es(son) verdadera(s) respecto de una reflexión?
I. El perímetro se mantiene inalterable.
II. El área no varía.
III. Es lo mismo que una simetría.
A) Sólo III B) Sólo I y II C) Sólo I y III D) Sólo II y III E) I, II y III
12. Se traslada el segmento AB de coordenadas A(-1, 2) y B(2, 4), hasta el segmento '
B
'
A de
coordenadas A’(-6, -2) y B’(-3, 0). El vector de traslación es
A) (4, 5) B) (4, 3) C) (5, 4) D) (-5, -4) E) (-4, -5)
13. El punto simétrico de P(4, 3) respecto al punto (2, -1) es
A) )
5
,
0
(  B) (6, 2) C) (0, 3) D) (-4, -3) E) (-5, 0)
14. ¿Cuál(es) de las siguientes figuras geométricas tiene(n) más de 2 ejes de simetría?
I) El rectángulo
II) El cuadrado
III) El rombo
A) Sólo II B) Sólo I y II C) Sólo II y III D) I, II y III E) Ninguna
15. El punto simétrico a P(-3, 2), con respecto al eje de las ordenadas, es
A) (-3, -2) B) (3, -2) C) (-3, 2) D) (3, 2) E) (2, -3)
16. El total de ejes de simetría que tiene un pentágono regular son
A) 2 B) 3 C) 4 D) 5 E) 6
17. Los vértices de un triángulo son A(0, 0), B(4, 0) y C(1, 6). Al rotar el triángulo en 270º con
respecto al origen, las coordenadas de C resultan
A) (6, -1) B) (-1, 6) C) (6, 1) D) (-1, -6) E) (1, -6)
18. Al trasladar el punto A(4, 6) hasta A’(-2 , 3), el vector de traslación es
A) (2, 8) B) (-6, -3) C) )
2
1
,
2
1
( D) (-8, 18) E) (-2, 2)
Danny Perich C.
www.sectormatematica.cl
19. Si el punto P(-2, 7) se traslada a P’(4, -1), según el vector de traslación (x – 1, y + 2).
Entonces x + y =
A) -3 B) -1 C) 0 D) 1 E) 2
20. Una circunferencia de centro (4, 2) y que contiene al punto (0, 0), rota 90º respecto al origen.
El nuevo centro está ubicado en
A) (-2, -4) B) (-2, 4) C) (-4, 2) D) (-4, -2) E) (4, -2)
21. ¿Cuál de las siguientes alternativas corresponde a una trasformación del punto (-2, -1) en el
punto (-1, 2)?
A) una traslación
B) una rotación en 90º respecto al origen
C) una reflexión
D) una rotación en 180º respecto del origen
E) una rotación en 270º respecto del origen
22. Trasladar el punto A(4, -3) al punto B(2, -1) y luego al punto C(-3,5), equivale a la traslación
única
A) (-7, 8) B) (3, 1) C) (1, 2) D) (-1, 4) E) (6, -4)
23. Al rotar el cuadrado de vértices A(1, 1), B(4, 1), C(1, 4) y D(4, 4) en 90º y luego trasladarlo
según el vector (1, -1), el vértice que queda sobre el origen es
A) D B) C C) B D) A E) Ninguno
24. La reflexión, respecto del origen, del punto medio del trazo cuyos extremos son A(6, 2) y
B(-4, 4) es
A) (1, 3) B) (1, -3) C) (-1, -3) D) (-1, 3) E) (1, 3)
25. ¿Cuál de las siguientes letras no tiene eje de simetría?
A) E B) H C) A D) Z E) O
26. Al punto (6, -4) se le aplica una traslación obteniendo el punto (12, -8). Si al punto (-3, 5) se
le aplica la misma traslación, entonces se obtiene el punto
A) (-6, 10) B) (-9, 9) C) (9, -3) D) (3, 1) E) (6, 9)
27. ¿Cuántos ejes de simetría tiene el romboide?
A) 0 B) 1 C) 2 D) 3 E) 4
28. ¿Cuál de las siguientes figuras no tiene centro de simetría?
A) Triángulo rectángulo B) Cuadrado C) Rectángulo D) Rombo E) Circunferencia
Danny Perich C.
www.sectormatematica.cl
29. Al trasladar el triángulo de vértices A(-1,5), B(2,0) y C(3,1), según el vector de traslación
(4,1), el vértice homólogo de B es
A) (3,6) B) (2,1) C) (6,0) D) (6,1) E) (7,2)
30. Una circunferencia tiene como centro el punto (3,5). Si el vector de traslación de este punto es
(-5, 1), ¿Cuál es el centro de la circunferencia trasladada?
A) (-2,6) B) (8,6) C) (-2,4) D) (-15,5) E) (8,4)
31. El triángulo que resulta al rotar, con centro en el origen y ángulo de 180º, el triángulo de
vértices: A = (2,3), B = (7,-2) y C = (5,8), tiene coordenadas
A) A = (2,3), B = (7,-2) y C = (5,8)
B) A = (-2,-3), B = (-7,2) y C = (-5,-8)
C) A = (3,2), B = (-2,7) y C = (8,5)
D) A = (3,-2), B = (-2,-7) y C = (8,-5)
E) A = (-2,3), B = (-7,-2) y C = (-5,8)
32. El vector traslación que transforma el punto (2,5) en el punto (-9,2), es
A) (11,3) B) (-7,3) C) (-7,-7) D) (-11,-3) E) (11,-3)
33. Si al triángulo ABC, de coordenadas A(2, 2), B(2, -4) y C(6, -1), se le aplica una rotación de
90º, con centro en el origen, y luego una traslación T(5,-2), el vértice C sería
A) (1,6) B) (6,4) C) (11,-3) D) (1,1) E) Otro
34. Dados los siguientes triángulos, determinar cuáles son congruentes.
I) II) III)
A) Sólo I B) Sólo I y II C) Sólo I y III D) Sólo II y III E) I, II y III
35. Un alumno para demostrar en el cuadrado de la figura que ABC  DBC, determinó que
 , que  y que el CAB  CDB, por ser rectos. ¿Qué criterio de congruencia
utilizó?
A) LLL
B) LAL
C) ALA
D) AAL
E) LLA
75°
8 cm.
65
°
75°
8 cm. 65
°
75°
8 cm.
65
°
C D
B
A
Danny Perich C.
www.sectormatematica.cl
36. En la figura siguiente, ABD  CDB y ADB  CBD. ¿Qué criterio de congruencia permite
demostrar que el ABD  CDB?
A) LLL
B) LAL
C) ALA
D) AAL
E) LLA
37. Dos triángulos son congruentes si
A) Tienen dos pares de ángulos respectivamente iguales
B) Tienen dos pares de lados respectivamente iguales
C) Tienen igual área
D) Tienen igual perímetro
E) Tienen sus lados respectivos en razón 1:1
38. Los lados de un triángulo rectángulo miden 6 cm. y 8 cm. El perímetro de un triángulo
congruente al anterior es
A) 10 B) 14 C) 24 D) 48 E) No se puede determinar
39. ¿En cuál(es) de las siguientes figuras se determina(n) dos triángulos congruentes al trazar una
de sus diagonales?
I. Cuadrado II. Rectángulo III. Rombo
A) Sólo I B) Sólo I y II C) Sólo I y III D) Sólo II y III E) I, II y III
40. Al trazar las diagonales de un cuadrado es falso que se formen cuatro triángulos
A) Congruentes B) Isósceles C) Rectángulos D) Equiláteros E) De igual área
41. En la figura, AC  DF y BC  EF . El valor de x es
A) 15º
B) 30º
C) 60º
D) 90º
E) 150º
42. En la figura, el CDE es isósceles. C es punto medio de y D es punto medio de . ¿Qué
criterio de congruencia permite demostrar que el ACE  BDE?
A) LAL
B) ALA
C) LLA
D) LLL
E) AAL
A D
C B
E
C
B
A
D
30 30
A
C
B
D
E
F
3x 2x
Danny Perich C.
www.sectormatematica.cl
43. En los triángulos siguientes se verifica que  , que  y que el CAB  FDE. ¿Qué
criterio permite demostrar que estos triángulos son congruentes?
A) LLL
B) LAL
C) ALA
D) LLA
E) Falta Información
44. En la figura, el ABC  DEF, siendo CAB  FDE y ACB  DFE, entonces se verifica que
A) AC  DF
B) BC  DE
C) AB  FE
D) AC  FE
E) AB  FD
45. Para demostrar que los triángulos AOB y COD de la figura, son congruentes, es necesario saber
que
A) AB  DC
B) BAO  DCO
C) AB // CD
D) AO  DO y AB  CD
E) BO  CO y AO  DO
46. Los triángulos ABC y DEF de la figura son congruentes, entonces la medida de es
A) 9
B) 15
C) 17
D) 40
E) Falta información
47. Marca la alternativa de la proposición verdadera
A) Dos triángulos rectángulos son congruentes si sus ángulos agudos respectivos son
congruentes.
B) Dos triángulos son congruentes si sus lados homólogos miden lo mismo.
C) Dos triángulos son congruentes si sus ángulos respectivos son iguales.
D) Para demostrar que dos triángulos son congruentes se puede utilizar el criterio AAL.
E) Todos los triángulos equiláteros son congruentes.
48. En la figura, ABC equilátero y   . El criterio que permite demostrar que los
triángulos AFE, BDF y CED son congruentes es:
A) LAL
B) LLL
C) ALA
D) LLA
E) LAA
A B
C D
F E
C
A B
D
F
E
B
O
A
D
C
C
E
D
F
B
A
A
C
17
B
40
80
15
F
D
E
60
80
Danny Perich C.
www.sectormatematica.cl
49. El área del rombo cuyas diagonales son x – 2 y x + 2 es
A) 4
x2
 B)
2
4
x2

C)
4
4
x2

D)
8
4
x2

E) No se puede determinar
50. El área de un triángulo equilátero de lado 6 es
A) 36 B) 18 C)
2
3
6
D) 3
9 E) Ninguna de las anteriores
51. Si la diagonal de un cuadrado mide 3 cm., entonces el radio de la circunferencia inscrita al
cuadrado es
A)
4
2
3
cm. B) 1,5 cm. C)
2
2
3
cm. D) 3 cm. E) Ninguna de las anteriores
52. El área de un triángulo equilátero es 3
18 cm2
, ¿cuál es su perímetro?
A) 2
18 cm. B) 3
6 cm. C) 3
18 D) 2
6 E) 3
54
53. Las diagonales de un rombo son 10 cm. y 24 cm. ¿Cuánto mide el área del cuadrado
construido sobre uno de los lados del rombo?
A) 240 cm2
B) 120 cm2
C) 52 cm2
D) 169 cm2
E) 26 cm2
54. Si el lado de un cuadrado aumenta 3 veces, entonces su área aumenta
A) 3 veces B) 6 veces C) 9 veces D) 12 veces E) 24 veces
55. Si la arista a de un cubo aumenta en 2 unidades, ¿en cuánto aumenta su área?
A) 2
)
2
a
(  B) 2
)
2
a
(
6  C) )
1
a
(
24  D) )
4
a
(
6 2
 E) Ninguna de las anteriores
56. La mediana de un trapecio mide 40 cm. Si la altura corresponde a los
4
3
de la mediana,
¿cuál es el área del trapecio?
A) 10 cm2
B) 900 cm2
C) 1.600 cm2
D) 600 cm2
E) 1.200 cm2
57. El radio de una circunferencia tiene la misma medida que la diagonal de un cuadrado de área
16 cm2
. El área del círculo es
A) 
4 cm2
B) 
16 cm2
C) 
32 cm2
D) 
8 cm2
E) 
2
4 cm2
58. Los lados de un triángulo equilátero y de un cuadrado están en razón 1 : 2. ¿Cuál es la razón
entre sus áreas?
A) 3 : 16 B) 3 : 4 C) 1 : 4 D) 1 : 2 E) Ninguna de las anteriores
Danny Perich C.
www.sectormatematica.cl
59. El lado de un cuadrado mide 4
log2 cm. ¿Cuántos cm2
mide su área?
A) 8
log2 B) 4
log4 C) 16
log4 D) 81
log3 E) Ninguna de las anteriores
60. Al rotar un cuadrado de diagonal 4 cm., con respecto a uno de sus lados, se obtiene una figura
cuya área es
A) 16 cm2
B) 8 cm2
C) 
16 cm2
D) 
8 cm2
E) 
2
16 cm2
61. La diferencia entre el área de un cuadrado de lado 10 m. y el área de un triángulo equilátero
de altura 3
10 m es
A) 10 – 10 B) 100-10 C) 100-25 D) 100-100 E) Otro valor
62. Los lados de un triángulo son a, 2a y 3a. Entonces su perímetro es
A) 5a B) 6a C) 3
a
5 D) 3
a
6 E) Falta Información
63. El área de un rectángulo de lados a y (a + b) es
A) 2a + b B) 4a + 2b C) b
a2
 D) ab
a
2  E) ab
a2

64. El área de un cuadrado es 4 cm2
. ¿Cuál es el perímetro del triángulo equilátero construido
sobre su diagonal?
A) 8 cm. B) cm. C) cm. D) cm. E) cm.
65. Con la quinta parte del perímetro de una circunferencia se construye una circunferencia de
16 cm. de longitud. ¿Cuál es el radio de la circunferencia mayor?
A) 8 cm. B) 16 cm. C) 20 cm. D) 40 cm. E) 80 cm.
66. El área de un cuadrado es 64 cm2
. Si cada lado disminuye a la cuarta parte, ¿cuánto mide la
mitad del área del cuadrado resultante?
A) 32 cm2
B) 16 cm2
C) 8 cm2
D) 4 cm2
E) 2 cm2
67. Si en un triángulo equilátero la longitud de cada lado aumenta en una unidad, entonces
¿cuál de las siguientes afirmaciones es verdadera?
A) su perímetro aumenta en 3 unidades
B) su área aumenta en 3 unidades cuadradas
C) su perímetro permanece constante
D) su área permanece constante
E) su altura aumenta en 1 unidad
68. La superficie de un cuadrado es 4x2
+ 4x + 1. Si el lado aumenta en 2 unidades, su área
aumenta en
A) 2 cm2
B) 4 cm2
C) (8x + 8) cm2
D) 8 cm2
E) 8x cm2
Danny Perich C.
www.sectormatematica.cl
69. Cada arista del cubo de la figura, mide 2 cm. ¿Cuánto mide la
superficie del cuadrilátero sombreado?
A) 4 cm2
B) 8 cm2
C) 16 cm2
D) 2
2 cm2
E) 2
4 cm2
70. La superficie de un cubo es 6x2
– 12x + 6, si la arista disminuye en 1
unidad, el área de una de sus caras es
A) 2
x B) 5
x
12
x
6 2

 C) 1
x
2
x2

 D) 4
x2
 E) 4
x
4
x2


71. En la circunferencia de centro O de la figura, CAO = 20º, AOB = 100º. El valor del OBC
es
A) 10º
B) 20º
C) 25º
D) 30º
E) 40º
72. En la circunferencia de centro O, AO
AB  . La medida del COA es 140º, entonces x mide
A) 40º
B) 50º
C) 90º
D) 130º
E) 140º
73. En la figura siguiente, OB es radio de la circunferencia. Si CAB = 70º y ABO = 25º,
entonces ACB =
A) 25º
B) 45º
C) 65º
D) 80º
E) 130º
74. Los arcos MN, NP y PQ de la circunferencia de centro O, son de igual medida. Si
MOP = 112º, la medida del PRQ es
A) 14º
B) 28º
C) 34º
D) 56º
E) 68º
A
B
O
C
A
x
B
O
C
A B
O
C
R
O
M
P
Q
N
Danny Perich C.
www.sectormatematica.cl
75. En la circunferencia de centro O y diámetro AD de la figura, los arcos AB, BC y CD están en la
razón 6 : 3 : 1. El valor de x es
A) 108º
B) 54º
C) 30º
D) 27º
E) 18º
76. AD y CD tangentes a la circunferencia de centro 0. Si BC
AB  y BC
AC  , entonces ADC
mide
A) 15º
B) 30º
C) 45º
D) 60º
E) 90º
77. En la semicircunferencia de centro O, OBC = 40º.
La medida del AOC es
A) No se puede determinar
B) 20º C) 40º D) 60º E) 80º
78. En la figura, OC
//
AB , AD diámetro de la circunferencia de
centro O. Si el ángulo BAD mide 40º, entonces el BOC mide
A) 20º
B) 40º
C) 60º
D) 80º
E) 100º
79. En la circunferencia de centro O siguiente, la medida del ángulo x es
A) No se puede determinar
B) 90º
C) 45º
D) 60º
E) 30º
80. En la circunferencia de centro O, AO
AB  . La medida del ACB es
A) 15º B) 30º C) 45º
D) 60º E) Faltan datos
81. A un cuadrado se le circunscribe una circunferencia de radio r. La razón entre sus áreas,
respectivamente es
A) 2
:
1 B) 2
:
 C) 4
:
 D) 
:
1 E) 
:
2
x
C
O
A
B
D
A
C
B
O
D
C
B
O
A
O
B
A
D
C
x
x
O
A B
C
O
.
Danny Perich C.
www.sectormatematica.cl
82. El área de un círculo es  , entonces su perímetro mide
A)
2

B)  C) 
2 D) 
4 E) Ninguna de las anteriores
83. Si el radio de una circunferencia y la diagonal de un cuadrado miden lo mismo, entonces la
razón entre sus respectivos perímetros es
A) 2
:
 B) 2
4
:
 C) 2
:
 D) 2
:
2 E) 2
2
:

84. El radio de la circunferencia circunscrita del hexágono regular de área 27 es
A)
2
3
B) 2 C)
2
2
D) 2
2 E) 2
85. El volumen de un cubo cuya área total es 12 cm2
es
A) 2 cm3
B) 8 cm3
C) 2 cm3
D) 2
2 cm3
E) 3
12 cm3
86. El volumen, en cm3
, de un cilindro de diámetro 10 cm. y altura 12 cm., es
A) 1200 B) 300 C) 240 D) 120 E) 120
87. Unas pelotas se venden en latas de forma cilíndrica que contienen 3 pelotas cada una. Si el
diámetro de la lata es de 6 cm. Calcular el volumen que queda sin ocupar en el interior de una
lata.
A) 18 cm3
B) 36 cm3
C) 54 cm3
D) 108 cm3
E) Ninguna de las anteriores
88. Una cúpula mide 8 m de diámetro, ¿cuál es su superficie si es semiesférica?
A) 32 B) 64 C)
3
128
 D)
3
256
 E)
3
64

89. Calcular el volumen, en cm3
, de una pirámide cuadrada de 6 cm de lado y altura de una cara
cm.
A) B) C) 96 D) 288 E) Ninguna de las anteriores
90. Determinar el volumen, en cm3
, de una superficie esférica de 6 cm. de diámetro.
A) 9 B) 12 C) 24 D) 36 E) 288
91. Calcular el volumen, en m3
, de un depósito cilíndrico de radio 3 m. y altura 4 m. terminado en
una semiesfera.
A) 54 B) 36 C) 30 D) 18 E) Ninguna de las anteriores
Danny Perich C.
www.sectormatematica.cl
92. Calcular el volumen generado por un triángulo equilátero de 2 cm. de altura al girar alrededor
de ésta.
A)
4
3
cm3
B)
64
3
3
cm3
C)
8

cm3
D)
16

cm3
E)
9
8
cm3
93. La diagonal de una de las caras de un cubo es 2
3 m. Calcular la superficie del cubo.
A) 9 cm2
B) 18 cm2
C) 2
18 cm2
D) 27 cm2
E) 54 cm2
94. El área de una esfera es 48 cm2
. Aproximando el valor de  a 3, determinar la medida de su
diámetro.
A) 2 cm. B) 4 cm. C) 8 cm. D) 16 cm. E) Ninguna de las anteriores
95. A Mario lo contratan para pintar un recipiente cilíndrico de 20 m de diámetro y 15 m de altura,
por el que cobra 750 pesos el metro cuadrado, ¿cuánto se le debe cancelar a Mario por el
trabajo hecho, aproximando el valor de  a 3?
A) $ 225.000 B) $ 675.000 C) $ 1.125.000 D) $ 1.350.000 E) $ 3.375.000
96. Si el largo de un paralelepípedo aumenta en un 25%, el ancho disminuye en un 20% y el alto
se mantiene constante, entonces el volumen resultante, respecto del volumen original
A) aumenta en 5% B) disminuye en 5% C) aumenta en 10%
D) se mantiene constante E) disminuye 10%
97. El perímetro basal de una pirámide recta de base cuadrada es 10m, si la altura de la pirámide
es 3m; entonces su volumen es
A) 10 m3
B) 3 m3
C) 6,25 m3
D) 27 m3
E) 9 m3
98. Si se tiene una esfera de volumen V cm.3
y área de A cm.2
, el radio de dicha esfera en función
de A y V es
A)
A
V
cm. B)
V
A3
cm. C)
V
A
3 3
cm. D)
3
2
A
V
3
cm. E)
A
V
3
cm.
99. La capacidad de un cubo es 8 litros. La suma de todas las aristas del cubo es
A) 160 cm. B) 200 cm. C) 220 cm. D) 240 cm. E) Otro valor
100. ¿Cuánto vale el volumen de un cono, si su altura es 2
3 y el perímetro de la base es 4 cm?
A) 2
3 B) 6
2 C) 3
3 D) 2
4 E) Otro valor
101. Dos cuadriláteros A y B son semejantes. Los lados del cuadrilátero A son 10, 15, 18 y 12 cm.
Si la constante de proporcionalidad es 3, ¿cuánto mide el menor de los lados de B?
A) 3
,
0 cm. B) 3
,
3 cm. C) 5 cm. D) 6 cm. E) 30 cm.
Danny Perich C.
www.sectormatematica.cl
102. Los perímetros de dos polígonos semejantes P y Q son 45 y 54. El lado mayor de P es 15,
¿cuál es el lado mayor de Q?
A) 5 B) 6 C) 15 D) 18 E) 24
103. En un triángulo las medidas de los ángulos interiores están en la razón 4 : 9 : 5.
El triángulo es
I) Isósceles II) Rectángulo III) Acutángulo
A) Sólo I B) Sólo II C) Sólo III D) Sólo I y II E) Sólo I y III
104. Calcular la altura de un árbol que proyecta una sombra de 4,2 metros, si se sabe que un
poste de 2,5 metros de altura proyecta, en el mismo momento, una sombra de 1,4 metros.
A) 2,35 m. B) 4,2 m. C) 5,3 m. D) 7,5 m. E) 15 m.
105. Los triángulos ABC y DEF son semejantes. = 6 cm., = 12 cm., = 10 cm. y
= 7,5 cm. Determinar + .
A) 7,2 cm.
B) 12,5 cm.
C) 19,5 cm.
D) 19,7 cm.
E) 24,5 cm.
106. Tres árboles se encuentran alineados. El más pequeño mide 2 metros, el mediano mide 3,5
metros. Si la distancia entre cada árbol es de 15 metros, ¿cuánto mide el árbol más alto?
A) 3,5 m. B) 5 m. C) 5,5 m. D) 7 m. E) 15 m.
107. Los lados de un polígono miden 6, 9, 12 y 15 cm. ¿Cuál es el perímetro del polígono
semejante al anterior si su lado mayor mide 20 cm?
A) 42 cm. B) 47 cm. C) 56 cm. D) 62 cm. E) Ninguna de las anteriores
108. La sombra de un edificio es de 50 metros y a esa misma hora la sombra de una casa de
5 metros de altura, es de 10 metros. ¿Cuál es la altura del edificio?
A) 10 m. B) 25 m. C) 45 m. D) 50 m. E) 100 m.
109. En un triángulo isósceles las medidas del ángulo de la base y del vértice están en la razón
1:3; el ángulo mayor mide
A) 36º B) 45º C) 90º D) 108º E) 135º
110. En un triángulo rectángulo los segmentos que la altura determina sobre la hipotenusa miden
16 y 36. El área del triángulo es
A) 39 B) 78 C) 108 D) 216 E) 624
F
E
D
C
A B
Danny Perich C.
www.sectormatematica.cl
111. En la figura // . Determinar la medida de si = 20 cm, = 6 cm. y = 18 cm.
A) 9 cm.
B) 11 cm.
C) 12,6 cm.
D) 54 cm.
E) 58 cm.
112. // ; = 15 cm., = 5 cm., = 3 cm., =
A) 13 cm.
B) 10 cm.
C) 9 cm.
D) 6 cm.
E) 1 cm.
113. En la figura, // , entonces
I)
CD
AC
AB
DE

II)
EC
BC
DE
AB

III)
CD
DE
AC
AB

A) Sólo I B) Sólo II C) Sólo III D) Sólo II y III E) I, II y III
114. En la figura, // , si = x + 3, = x + 2, = x + 8, = x + 6.
La expresión que permite determinar x es
A)
6
x
8
x
2
x
3
x





B)
3
x
8
x
6
x
2
x





C)
6
x
14
x
2
5
x
2
2
x





D)
14
x
2
8
x
3
x
5
x
2





E) 14
x
2
5
x
2 


115. ABMN trapecio. = 8 cm, = 12 cm, = 15 cm. Entonces mide
A) 22,5 cm.
B) 11 cm.
C) 10 cm.
D) 6,4 cm.
E) Ninguna de las anteriores
58°
C
D
A
B
E
58°
A
D
B
E
C
A
D
B
E
C
C
N M
B
A
S
T R
Q
P
Danny Perich C.
www.sectormatematica.cl
116. ABCD es paralelogramo, = 12, = 4, = 18. Determinar .
A) 54
B) 4
C) 6
D) 9
E) 12
117. En la figura, = a, = b, = c. Si // , entonces queda determinado por la
expresión
A)
c
)
b
a
(
a 
B)
b
a
ac

C)
a
bc
D)
b
ac
E)
c
ab
118. En la figura, // , entonces
I)
OP
MO
PQ
MN

II)
OQ
ON
OP
OM

III) 2
OQ
NO
MN 

IV) MN
PQ
PQ2


A) Sólo I B) Sólo II C) Sólo II y III D) Sólo II y IV E) Sólo I y II
119. ¿Cuál(es) de las siguientes relaciones se verifica(n) en la figura, siendo // y // ?
I)
CF
AC
BE
AB

II)
EF
BC
AE
AB

III)
AF
AE
AC
AB

A) Sólo I
B) Sólo II
C) Sólo III
D) Sólo I y II
E) I, II y III
D
E
C
B
A
N
Q
M
P
O
A
B
E
C
F
D
C
F
D
E
A B
Danny Perich C.
www.sectormatematica.cl
120. Si // , = 5, = 8, = 6, el valor de es
A) 15,6
B) 9,6
C) 9
D) 6,6
E) 3,7
121. En la figura, ∆ABC  ∆DEF. Si  ; = = 5 cm. y = 4 cm. ¿Cuánto mide ?
A) 2 cm.
B) 3 cm.
C) 4 cm.
D) 5 cm.
E) 6 cm.
122. Los triángulos ABC y DEF de la figura son congruentes, entonces la medida de es
A) 8
B) 12
C) 15
D) 24
E) 30
123. En la figura siguiente, AC y BC son tangentes a la circunferencia de centro O.
Si ACB = 70°, entonces el ABO =
A) 20º
B) 35º
C) 45º
D) 55º
E) 70º
124. Desde un punto distante 5 cm. del centro de una circunferencia se ha trazado a ésta una
tangente de 3 cm. de longitud. Determinar la medida del diámetro de la circunferencia.
A) 2,5 cm. B) 4 cm. C) 5 cm. D) 8 cm. E) 10 cm.
125. En una circunferencia de centro O, se trazan los radios y , formando la cuerda ,
distinta al diámetro. Si AOB : BAO = 1 : 2. ¿Cuánto mide el ángulo inscrito en el arco
AB?
A) 18º B) 22,5º C) 36º D) 45º E) 72º
O
A
C B
O
A
C
P
M
L
D
B
A
C
B
F
D E
100
50
100
30
8
12
15
A
B
C
D E
F
G
Danny Perich C.
www.sectormatematica.cl
126. En la circunferencia siguiente, = 3, = 12 y = 13. Calcular el menor valor del
segmento .
A) 1
B) 3,25
C) 4
D) 6,5
E) 9
127. Se tienen dos circunferencias concéntricas de radios 5 y 9 cm. En la circunferencia mayor,
se traza la cuerda que intersecta a la menor en los puntos B y C de manera que
= = . El valor de es
A) 4 B) 8 C) 12 D) 16 E) Falta Información
128. = 2 cm., = 6 cm. y = 8 cm. El valor de en la siguiente circunferencia es:
A) 40
B) 24
C) 4
D) 6
,
2
E) 1,5
129. Dos cuerdas se interceptan en una circunferencia formando trazos de medidas 2 y a + 8 en
una de las cuerdas y medidas a + 3 y a + 2 en la otra. La cuerda mayor mide
A) -5 B) 2 C) 9 D) 12 E) 20
130. Los radios de tres circunferencias tangentes exteriores miden 2, 4 y 6 cm, respectivamente.
El área del triángulo que se forma al unir sus centros es
A) 16 cm2
B) 18 cm2
C) 20 cm2
D) 24 cm2
E) No se puede determinar
131. En la figura, el segmento tangente mide 8 cm. y los segmentos exterior e interior,
determinados por la secante miden 4 cm. y x cm., respectivamente. La medida de x es
A) 2
B) 4
C) 12
D) 16
E) Ninguna de las anteriores
132. Una circunferencia está inscrita en el triángulo ABC, siendo P, Q y R los puntos de tangencia.
¿Cuánto mide el segmento de tangente CQ, sabiendo que el perímetro del triángulo es
40 cm, que = 7 cm. y = 15 cm?
A) 5 B) 6 C) 8 D) 10 E) 12
A
P
D
C
B
P
A
B
C
D
Danny Perich C.
www.sectormatematica.cl
133. En la figura siguiente = 3 m. y = 5 m., el valor de es
A)
3
16
m. B)
3
4
m. C)
3
25
m.
D) 2
5 m. E) 3
2
5  m.
134. Los catetos de un triángulo rectángulo miden 3 cm. y 4 cm. Determinar la proyección mayor
de los catetos sobre la hipotenusa.
A) 1,8 cm. B) 3,2 cm. C) 4 cm. D) 5 cm. E)
2
5
cm.
135. En la figura siguiente, = 6 cm.; = 3 cm. Determinar el área del triángulo ABC.
A) 9 cm2
B) 12 cm2
C) 15 cm2
D) 18 cm2
E) 45 cm2
136. La altura hc de un triángulo ABC, rectángulo en C, es de 4 metros. Si los segmentos
determinados sobre la hipotenusa están en la razón 1:2, ¿cuánto mide el área del triángulo
ABC?
A) 2 m2
B) 2
2 m2
C) 2
4 m2
D) 2
6 m2
E) 2
12 m2
137. En un triángulo ABC, rectángulo en C, se traza la altura . Si = 3,2 m. y = 5 m.;
el valor de es
A) 1,8 m. B) 3 m. C) 4 m. D) 76
,
5 m. E) 16 m.
138. En la figura, = 5-1
cm; = 2-1
cm; la altura del triángulo ABC es
A)
10
1
B)
10
10
C) 10
D) 10 E) Ninguna de las anteriores
139. Los catetos de un triángulo rectángulo miden 3 cm. y 4 cm. Determinar la altura sobre la
hipotenusa de este triángulo.
A)
5
9
cm. B)
5
12
cm. C)
5
16
cm.
D) 5 cm. E) Ninguna de las anteriores
C
D B
A
C
D B
A
C
D B
A
Danny Perich C.
www.sectormatematica.cl
140. = 12 cm; = 9 cm; =
A) 3
3 cm.
B) 6 cm.
C) 3
6 cm.
D) 36 cm.
E) Ninguna de las anteriores
141. Los catetos de un triángulo rectángulo están en la razón 3 : 4. Si la hipotenusa mide 10 cm.,
entonces el cateto menor mide
A) 2 cm. B) 3 cm. C) 3,6 cm. D) 6 cm. E) 8 cm.
142. = 10 cm; = (p + 2) cm; = 2p cm; = ?
A) 3,6 cm.
B) 4 cm.
C) 4,8 cm.
D) 6,4 cm.
E) 22,04 cm.
143. En la figura siguiente, AB
//
DE , 3
:
2
AD
:
CD  y 24
AB  , entonces la medida de segmento
DE es igual a
A) 8
B) 9,6
C) 12
D) 15
E) 16
144. En la figura, O es el centro de la circunferencia circunscrita al triángulo ABC y 3
OD  .
¿Cuánto mide el radio de la circunferencia circunscrita al triángulo ABC?
A) 2
B) 3
C) 1
D) Falta información
E) Otro valor
145. En la figura, AF = 40 cm. ha sido dividido en distintas partes. AB = 10 cm.,
AD = 2 AB – 3 cm. y DE = EF + 5 cm. ¿Cuánto mide EF ?
A) 7 cm.
B) 8 cm.
C) 9 cm.
D) 14 cm.
E) 18 cm.
A B
C
D
O
60
o
3
A B D E F
C
D B
A
C
D B
A
C
D E
A B
Danny Perich C.
www.sectormatematica.cl
146. En un triángulo cualquiera ABC se traza la bisectriz . Si = c, = b y = a.
¿Cuál es el valor de ?
A) ac B)
2
ac
C)
b
ac
D)
c
ab
E)
c
b
a2
147. En la figura: ABC es equilátero y DCB es recto. ¿Cuál(es) de las siguientes afirmaciones
es(son) verdadera(s)?
I. 2· = +
DAC es isósceles
III. 2
= 2
+ 2
A) Sólo I y II
B) Sólo I y III
C) Sólo II y III
D) I, II y III
E) Ninguna de ellas
148. ¿Cuál es la altura del trapecio isósceles si = 4 cm., = 10 cm. y = 12 cm?
A) 4 cm.
B) 3
4 cm.
C) 6 cm.
D) 3
6 cm.
E) 8 cm.
149.  , ABC = 90º, = 6, = 4, = 3. ¿Cuál es el valor de en la siguiente figura?
A) 9
B) 5
C)
9
5
D)
5
9
E) Falta información.
150. Determine la medida de una cuerda , sabiendo que O es centro de la circunferencia de
radio 12 cm. y que la distancia del centro a la cuerda es de 9 cm.
A) 3
3 cm. B) 7
3 cm. C) 7
6 cm. D) 15 cm. E) 30 cm.
151. Las circunferencias de centros P y Q son congruentes de radio 3 cm. cada una. ¿Cuánto mide
PQ si OP
2
AB
3 
A) 8 cm. B) 10 cm. C) 12 cm.
D) 15 cm. E) 18 cm.
D B
C
A
A B
D C
A
C
B
D E
P Q
A B
Danny Perich C.
www.sectormatematica.cl
152. En el círculo de la figura el diámetro es perpendicular a la cuerda . ¿Cuál es la medida
de , si el radio de la circunferencia mide 12 cm. y = 8 cm?
A) 12 cm.
B) 8 cm.
C) 2
8 cm.
D) 2
4 cm.
E) 4 cm.
153. En la figura // // . Si = 12 cm., = 8 cm., = 10 cm. ¿Cuál es el valor de ?
A) 12
B) 13
C) 14
D) 15
E) Ninguna de las anteriores.
154. En un triángulo rectángulo, los trazos que la altura determina sobre la hipotenusa miden 8 y
18 cm. Entonces el área del triángulo es
A) 78 cm2
B) 156 cm2
C) 312 cm2
D) 624 cm2
E) Ninguna de las anteriores
155. En el ABC de la figura, = , // ; // ; = 4 cm, = 2 cm; + =
A) 3
B) 6
C) 4
D) Falta información
E) Ninguna de las anteriores.
156. En el cuadrilátero ACBD, AB es bisectriz del  CAD y CD
AB  . Si
= = 6 cm. y = 4 cm. ¿Cuánto mide ?
A) cm. B) 8 cm. C) 7 cm.
D) 4 cm. E) Ninguna de las anteriores.
157. En la circunferencia de centro O y radio r, los triángulos MNO y
MNT son isósceles congruentes. Entonces, =
A)
2
3
r
B) 3
r
C) 3
r
2
D) 2
r
E) 2
r
2
A
D
B
C
E
F
E
D
C
B
A
C
D E
A
F
B A
B
C D
E
B
O
M
T
N
Danny Perich C.
www.sectormatematica.cl
158. En la figura, PT es tangente en T a la circunferencia de centro O y radio 6 cm. si = ,
el arco TQ mide
A) 12cm.
B) 9cm.
C) 6cm.
D) 3cm.
E) 2cm.
159. En la figura se tiene que // ; = 3 ; =16; =
A) 10
B) 12
C) 14
D) 16
E) 18
160. Dados dos lados de un triángulo miden 8 y 12 cm., entonces, el tercer lado puede medir
I. 4 II. 8 III. 24
A) Sólo I B) Sólo II C) Sólo III D) Sólo I y II E) Sólo I y III
161. ABC y BDE son triángulos equiláteros congruentes de lado 8. Si =4 , ¿cuánto mide ?
A) B) C)
D) E)
162. En un triángulo ABC rectángulo en C cuya hipotenusa mide p, la medida de la proyección de
un cateto sobre ella es m. ¿Cuál de las siguientes expresiones siempre representa al
cuadrado de la medida del otro cateto?
A) pm B) p2
- m2
C) (p - m)2
D) (pm)2
E) p2
- pm
163. Se tiene que = a; = 2a – 17; = a – 15 y = a + 2. Para que 1
L sea paralela a
2
L , el valor de AD debe ser
I) 5
II) 7
III) 15
A) Sólo I B) Sólo II C) Sólo III
D) Sólo I y II E) Sólo I y III
P
Q O
T
A
B C
D E
A
F
B
C
D
E
A
L1
L2
D E
B C
Danny Perich C.
www.sectormatematica.cl
164. ¿Cuánto vale la base de un trapecio cuya mediana vale 3x y su otra base vale 2x?
A) 4x B) x C) 6x D) 5x E) Otro valor
165. Desde un punto se traza un segmento tangente a una circunferencia de 16 cm. y
una secante, cuyo segmento exterior mide 8 cm. El segmento interior de esta secante mide
A) 24 cm. B) 31 cm. C) 96 cm. D) 192 cm. E) Ninguna de las anteriores
166. Un trazo AB está dividido interiormente en la razón 4 : 9, si el menor de los trazos mide
32 cm. ¿Cuál es la longitud del trazo?
A) 32 B) 72 C) 104 D) 108 E) 120
167. L1 // L2. Si = 2x; = x + 2; = 4x – 1 y = 2x + 3; el valor de x es
A) 2
B) 7
C) 4
D) -2
E) -1
168. Si en un triángulo rectángulo, p y q son las medidas de cada cateto respectivamente y r es la
longitud de la hipotenusa, entonces siempre ocurre que
A) p > r B) q > r C) r > p + q D) p < q E) q < r
169. Dado el triángulo ABC, de lados = 13; = 5 y = 12, entonces la altura desde el
vértice C mide
A)
13
30
B) 3 C) 4
D)
13
60
E) 5
170. La medida de los lados de un triángulo ABC son 6 cm., 8 cm. y 10 cm. El área del triángulo
formado por los puntos medios de los lados respectivos es
A) Un cuarto del área del triángulo ABC
B) La tercera parte del triángulo ABC
C) La mitad del triángulo ABC
D) El doble del área del triángulo ABC
E) No se puede determinar
171. La base de un triángulo y su altura están en razón 1 : 2. Si la base mide 16 cm. ¿Cuánto
medirá el lado de un cuadrado de área equivalente?
A) 8 cm. B) 16 cm. C) 64 cm. D) 32 cm. E) 24 cm.
A
B
C
A
L1
L2
D E
B C
Danny Perich C.
www.sectormatematica.cl
172. En un triángulo rectángulo, un cateto es el doble del otro y su perímetro es )
5
3
(
4  .
¿Cuál es su área?
A) 32 cm2
B) 16 cm2
C) 5
4 cm2
D) )
5
3
(
4  cm2
E) Faltan datos
173. En el triángulo rectángulo de la figura se verifica que
A) CD
AB
AC
2


B) BD
AD
BC
2


C) AB
BC
AC 

D) BC
AC
CD
2


E) BD
AB
BC
2


174. Si = , = 16, = 4. Entonces el área de la circunferencia es
A) 5
B) 10
C) 20
D) 25
E) 2,5
175. ¿Cuál(es) de las siguientes afirmaciones es(son) verdadera(s), de acuerdo a la figura
siguiente?
I)  = 
II)  = 
III)  = 
A) Sólo I B) Sólo II C) Sólo III
D) Sólo I y II E) Sólo II y III
176. El área de un rectángulo es 10 m2
; el largo es el doble del ancho. La medida de una de sus
diagonales es
A) 10 m B) 5 m C) 25 m D) 10 m E) 4 m
177. En el círculo de la figura CD
AB . ¿Cuál es la medida de CE, si el radio de la circunferencia
mide 12 cm. y cm
8
BE  ?
A) 12 cm.
B) 8 cm.
C) cm
2
8
D) cm
2
4
E) 4 cm.
A
E
B
D
C
A
O C
B
E
D
O
A B
C
D
E
C
D B
A
Danny Perich C.
www.sectormatematica.cl
178. La diagonal de un cuadrado mide 10 cm. Al disminuir su lado en 2
3
2
cm., la diagonal
disminuye en
A) 3 cm. B) 12 cm. C)
3
3
10
cm. D)
3
26
cm. E)
3
4
cm.
179. La suma de los ángulos interiores de un heptágono es
A) 360º B) 1080º C) 900º D) 1260º E) 1440º
180. En la figura, la tangente mide 6 cm. y la secante = 18 cm, entonces mide
A) 16 cm.
B) 6 cm.
C) 9 cm.
D) 4 cm.
E) 2 cm.
181. En la figura, se tiene que BCDE es un trapecio. Si = x + 4; = 2x – 2; = x y
= x + 6; entonces, el valor de x es
A) 2
B) 4
C) 6
D) 12
E) Otro valor
182. La medida de un ángulo exterior de un polígono regular de 18 lados es
A) 10º B) 20º C) 30º D) 40º E) 50º
183. En el triángulo ABC de la figura, las transversales de gravedad CE
y
AD se interceptan en
ángulo recto. Si GD =3 y GE=2, entonces BC mide
A) 13
2
B) 17
2
C) 18
2
D) 10
E) 8
184. El perímetro de un hexágono inscrito en una circunferencia es 30 cm. Al calcular el área de la
circunferencia resulta
A) 30 2
cm
 B) 36 2
cm
 C) 10 2
cm
 D) 5 2
cm
 E) 25 2
cm

R
P
B A
A E B
D
C
G
A
L1
L2
D E
B C
Danny Perich C.
www.sectormatematica.cl
185. En la circunferencia de centro O y radio r, MN es diámetro, si MP =r y Q punto medio de MP ,
entonces QN=
A) 3
r
B)
2
3
r
C)
2
13
r
D) 21
r
E) No se puede determinar
186. En la figura, 1// 2. Si AC
4 = CE, entonces BC con DE , respectivamente, están en la razón
A) 1 : 4
B) 1 : 5
C) 4 : 1
D) 5 : 1
E) ninguna de las anteriores
187. En la figura, ABC rectángulo en B, altura; además, // . Si AB = 2
5 cm. y
AM = 5 cm., entonces DC =
A) 5 cm.
B) 2
5 cm.
C) 10 cm.
D) 2
10 cm.
E) 2
25 cm.
188. Desde un punto situado a 40 cm. del centro de una circunferencia de 48 cm. de diámetro, se
traza una tangente. ¿Cuál es su longitud?
A) 8 B) 8 6 C) 4 D) 32 E) otro valor
189. Si dos polígonos son semejantes. Entonces ¿cuál(es) de las siguientes proposiciones es (son)
verdadera(s)?
I. Los polígonos tienen ángulos correspondientes de igual medida.
II. Sus lados correspondientes son proporcionales.
III. Los polígonos tienen la misma forma.
A) sólo I B) sólo I y II C) sólo I y III D) sólo II y III E) Todas
190. En la figura se tiene = 3; = 3,5; = 4; = 1,5; el
∆AMN ~ ∆ABC. ¿Cuál es el perímetro del ∆ABC?
A) 15,75 B) 13,25 C) 14,5
D) 14,55 E) Otro valor
A
B C
D E
L1
L2
M P
N
Q
O
A C
B
D
M
Danny Perich C.
www.sectormatematica.cl
191. Un árbol da una sombra de 3 metros y, a la misma hora, un mástil de 4 metros de altura
proyecta una sombra de 2 metros. ¿Cuál es la altura del árbol?
A) 2 m. B) 3 m. C) 6 m. D) 12 m. E) otro valor
192. Si los triángulos de la figura son semejantes, entonces el perímetro y área del triangulo A’B’C’
son respectivamente
A) 18 cm. y 13,5 cm2
B) 12 cm. y 6 cm2
C) 8 cm. y 4 cm2
D) 8 cm. y
3
8
cm2
E) 18 cm. y 9 cm2
193. ¿Cuál de las siguientes afirmaciones no es verdadera?
A) Dos polígonos congruentes son siempre semejantes.
B) Dos triángulos equiláteros son siempre semejantes.
C) Dos cuadrados son siempre semejantes.
D) Dos círculos son siempre semejantes.
E) Dos rectángulos son siempre semejantes.
194. Los lados de un triangulo miden 5, 6 y 8 cm. ¿Cuánto miden los lados de un triangulo
semejante si su lado más grande mide 16 cm?
A) 6, 7 y 16 cm. B) 14, 15 y 16 cm. C) 10, 12 y 16 cm.
D) 9, 12 y 16 cm. E) otras medidas.
195. Una fotografía de 14 cm. de largo por 10 cm. de ancho esta puesta en un marco que mide 2
cm. por lado más que la foto. ¿Cuál es la razón de semejanza entre el área de los rectángulos
que forman el marco y la fotografía?
A) 3 : 4 B) 5 : 4 C) 7 : 5 D) 9 : 5 E) otra razón
196. Una niña que mide 1 m proyecta una sombra de 2 m de largo. Si a esa misma hora y en ese
mismo lugar, un árbol proyecta una sombra de 8 m de largo, ¿cuál es la altura del árbol?
A) 4 m B) 5 m C) 6 m D) 7 m E) 16 m
197. Dos triángulos semejantes tienen áreas de 144 cm2
y 81 cm2
. La base del triángulo mayor
mide 30 cm. ¿Cuánto mide la base triangulo menor?
A) 22,5 cm. B) 16,875 cm. C) 25 cm. D) 24 cm. E) Otro valor.
198. Si ABC DEF, donde es homólogo con , = a cm y = 3a cm, ¿cuál de las
siguientes afirmaciones es siempre verdadera?
A) Si el área del triángulo ABC es 16 cm2
, entonces el área del triángulo DEF es 48 cm2
.
B) 3∙ABC = DEF
C) El perímetro del triángulo ABC es un tercio del perímetro del triángulo DEF.
D) // , // y //
E) Ninguna de las anteriores.
Danny Perich C.
www.sectormatematica.cl
199. Para que valor de x se tiene que 1 // 2?
A) 6,5
B) 8
C) 6
D) 5
E) 4
200. Dado un triangulo ABC, se dibuja una recta paralela a que corta en D y a en E. Los
segmentos y se cortan en X. ¿Cuál criterio de semejanza puede utilizar para probar de
la forma mas simple que los triángulos DXE y BXC son semejantes?
A) AA B) ALA C) LLA D) LAL E) No son semejantes.
201. En la figura 1 // 2, entonces se cumple
A)
d
b
c
a

B)
c
b
d
a

C)
d
c
b
a

D) ad = bc
E)
b
c
d
a

202. En la figura, si el ángulo en B es recto,
¿Cuál es la medida de x?
A) 12,25 B) 11 C) 11,25
D) 11,75 E) 12,75
203. Si ∆ABC ~ ∆DEF, ¿Cuál(es) de las siguientes expresiones es (son) equivalente(s) a ?
I)
EF
DE
BC 
II)
BC
AC
EF 
III)
DF
DE
AC 
A) sólo I B) sólo II C) sólo III D) sólo I y II E) sólo I y III
204. ¿A qué distancia del extremo P debe estar el punto A para que se cumpla : = 7 : 9;
si = 36 cm?
A) 18, 5 cm.
B) 20 cm.
C) 24 cm.
D) 25 cm.
E) 28 cm.
Danny Perich C.
www.sectormatematica.cl
205. ¿Cuánto vale x en la figura?
A) 6,25
B) 16
C) 3,5
D) 7
E) 4
206. Los perímetros de dos figuras semejantes son 30 cm. y 18 cm. ¿En qué razón están los
lados?
A) 25 : 9 B) 10 : 9 C) 6 : 2 D) 5 : 3 E) 5 : 2
207. En la figura, ¿Cuál es el valor de x, sabiendo que los ángulos a y b son congruentes?
A) 40
B) 22,5
C) 8
D) 8,4
E) 21
208. En la figura, = ; = 3; = 12; // . ¿Cuánto mide ?
A) 15
B) 18
C) 16
D) 21
E) 20
209. En la figura 1 // 2 // 3 // 4, si = 60, entonces la medida es
A) 12,75
B) 18,75
C) 11,25
D) 18,785
E) 30,75
210. En la figura, el área del triangulo ABC es 90 cm2
y // . ¿Cuál es el área del trapecio
ABED?
A) 36 cm2
B) 40 cm2
C) 50 cm2
D) 54 cm2
E) 60 cm2
Danny Perich C.
www.sectormatematica.cl
211. En los triángulos ABC y DEF de la figura, se sabe que // ,
// , = = 4, = = 8, y = 6, entonces el área del
triangulo ABC es
A) 180
B) 120
C) 108
D) 72
E) 54
212. Si =16; = 4; entonces =
A) 8
B) 48
4
C) 3
4
D) 3
8
E) 2
8
213. Los lados de un triangulo están en la razón 2 : 3 : 5 y su perímetro mide 55 cm. ¿Cuánto
mide el lado menor del triángulo?
A) 12 cm. B) 11 cm. C) 10 cm. D) 9 cm. E) 7,5 cm.
214. En el triangulo ABC de la figura, es bisectriz del ángulo ACB. Entonces el perímetro de este
triangulo es
A) 57 cm.
B) 51 cm.
C) 60 cm.
D) 15 cm.
E) 81 cm.
215. En la figura, los puntos P, Q, R y S están sobre la circunferencia de centro O.
Si : = 3 :4, = 6 , es el triple de , entonces mide
A) 4
B) 6
C) 8
D) 9
E) 10
216. En la figura, el lado del ∆ABD es el diámetro de la circunferencia de centro O. Para el
punto E en el lado , se tiene que = 3, = 12 y = 6. El valor del radio es
A)
2
270
B) 270 C)
2
352
D)
2
352
E)
2
252
Danny Perich C.
www.sectormatematica.cl
217. En la circunferencia de diámetro = 15 cm., = 3 cm. y = 4 cm. ¿Cuál es la longitud
de la cuerda ?
A) 13 cm.
B) 15 cm.
C) 7 cm.
D) 15,25 cm.
E) 9 cm.
218. ¿Qué significa que dos triángulos sean semejantes?
A) Que tienen igual área.
B) Que tienen igual perímetro.
C) Que sus lados son proporcionales.
D) Que sus tres lados respectivos coinciden
E) Que sus ángulos son proporcionales, en razón distinta de uno.
219. Según la figura ¿cuál(es) de las siguientes afirmaciones es (son) cierta(s)?
I) ΔACD ~ ΔCBE II) ΔBEC ~ ΔAEB III) ΔACD ~ ΔCAB
A) Sólo I
B) Sólo II
C) Sólo III
D) Sólo I y III
E) I, II y III
220. En la figura = 12, = 4, AT es tangente a la circunferencia de centro O, entonces
mide
A) 36
B) 32
C) 18
D) 16
E) 8
221. En la figura, si = 1 cm. y = 6 cm., entonces ¿cuánto mide ?
A) 5 cm.
B) 6 cm.
C) 26 cm.
D) 6 cm.
E) 25 cm.
Danny Perich C.
www.sectormatematica.cl
222. En el ΔABC de la figura, se sabe que = 90 cm., = 12 cm. // // y
: : = 1 : 2 : 3, entonces el valor de es
A) 96 cm.
B) 72 cm.
C) 48 cm.
D) 36 cm.
E) 24 cm.
223. En la figura es un diámetro de la circunferencia de centro O y radio r. es tangente en P
y mide r. Si M es el punto medio de , entonces la longitud de , en términos de r, es
A) r
B)
2
5
r
C)
2
3
r
D)
2
2
r
E)
3
4r
224. C es el punto de tangencia de con la circunferencia. Para ambas figuras el cálculo correcto
de x esta dado por
A) DB
AD
x 

2
B)
2
2
2
DB
AD
x 

C)
2
2
2
BD
BC
x 

D) BA
AD
x 

2
E)
2
2
2
BD
BC
x 

225. Si en la circunferencia de diámetro 30 cm. de la figura, la distancia
desde el centro O de ella, hasta la cuerda es de 9 cm., entonces la
cuerda mide
A) 6 cm. B) 12 cm. C) 18 cm.
D) 20 cm. E) 24 cm.
226. En la circunferencia de la figura se han trazado dos cuerdas con las medidas que se indican.
La relación correcta es
A) x = y B) x: y = 1: 3 C) y
x
2
1

D) xy = 75 E) x + y = 20
Danny Perich C.
www.sectormatematica.cl
227. Desde un punto situado a 17 cm. del centro de una circunferencia de 8 cm. de radio se dibuja
una tangente a la circunferencia, ¿Cuánto mide la tangente?
A) 17 cm. B) 2
6 cm. C) 12 cm. D) 5 cm. E) ninguna de las anteriores.
228. La tarjeta de la figura está dividida en cuatro partes, cada una de ellas semejante a la tarjeta
original. El valor de x es
A) 5
B) 10
C) 15
D) 20
E) 30
229. En una circunferencia de centro O, el diámetro = 2 cm. R es un punto fuera de ella. S es la
intersección de con la circunferencia y es tangente a ella. Si = , entonces mide
A) 3 cm. B) 6 cm. C) 2 cm. D) 2
6 cm. E) 2
3 cm.
230. En la figura se tiene que es diámetro. La medida del arco CA es el doble de la del arco
BC. Si mide 6 cm. ¿Cuántos centímetros cuadrados de área tiene el triángulo ABC?
A) 9
B) 18
C) 3
9
D) 3
18
E) 36
231. En la figura 1 // 2, PQ = 6 cm.  L2. El área del triangulo PQS es 36 cm2
. mide
A) 12 cm.
B) 18 cm.
C) 6 cm.
D) 9 cm.
E) no se puede determinar.
232. En la figura el triángulo ABC es rectángulo en A. = 10, = 6, = ,  ,
// . ¿Cuál es la medida de ?
A)
3
20
B) 8
C) 15
D) 13
2
E) 6,5
Danny Perich C.
www.sectormatematica.cl
233. En la figura // // ; // ; = 8; = 18.
Entonces =
A) 13
B) 10
C) 11
D) 12
E) 15
234. En la figura ABCD es cuadrado de área 144 cm2
y EFCD es rectángulo de área 36 cm2
. ¿Cuál
es el perímetro del triangulo AFE?
A) 30 cm.
B) 18 cm.
C) 27 cm.
D) 36 cm.
E) 108 cm.
235. En el trapecio ABCD de la figura está formado por el triangulo equilátero AED y el rombo
EBCD. Si la altura del triangulo es de 3
3 , ¿Cuál es el área del rombo?
A) 9 cm2
B) 3
9 cm2
C) 13,5 cm2
D) 3
18 cm2
E) 81 cm2
236. En la figura ABC es triangulo rectángulo en C, // , // y = 10 cm. Si
AB
5
4
BC  y : = 1 : 2. ¿Cuál es el área de la región achurada?
A) 24 cm2
B) 8 cm2
C) 6 cm2
D) 12 cm2
E) 13 cm2
237. ¿Cuál es el área del la región achurada del triangulo rectángulo en C de la figura si es
altura, = 29 cm. y = 4 cm?
A) 6 cm2
B) 8 cm2
C) 16 cm2
D) 18 cm2
E) 20 cm2
238. En la figura = 10 cm., = 4 cm. ¿Cuántos centímetros cuadrados de área tiene el
triángulo DBC?
A) 72,5 B) 125 C) 20
D) 40 E) 80
Danny Perich C.
www.sectormatematica.cl
239. En el triangulo ABC, // . Si = x + 4; = x + 6; = x y = x + 1. ¿Cuál es el
valor de x?
A) 4
B) 3
C) 2
D) 1
E) otro valor
240. En el triangulo ABC, = 10 y = 4, ¿en qué razón están las áreas de los triángulos ADC y
ABC?
A) 2 : 3 B) 2 : 5 C) 3 : 7
D) 3 : 2 E) 3 : 5
241. En la circunferencia de centro O y radio 12 cm de la figura, = 5 cm. ¿Cuánto mide el
segmento AC?
A) cm
B) cm
C) 7 cm
D) cm
E) Indeterminable con los datos dados.
242. La circunferencia de centro O de la figura tiene diámetro 20 cm. Si = 4 cm. ¿cuál es el
área del triangulo AOB?
A) 20 cm2
B) 48 cm2
C) 40 cm2
D) 30 cm2
E) 96 cm2
243. En la figura O es centro de la semicircunferencia. Si = y  . ¿Cuál(es) de las
siguientes proposiciones es (son) verdadera(s) considerando que = r?
I. = r
II. = 3
2
r
III. CBD = 2CDB
A) Sólo I B) Sólo III C) Sólo I y II
D) Sólo I y III E) I, II y III
Danny Perich C.
www.sectormatematica.cl
244. En el triángulo ABC de la figura, // , = 45 cm., = 5 cm. y : = 4 : 5. ¿Cuál
es el valor de ?
A) 30 cm.
B) 36 cm.
C) 45 cm.
D) 20 cm.
E) 25 cm.
245. Si en la circunferencia de centro O de la figura, = , con y diámetros, entonces la
medida del ángulo ACD es
A) 45º
B) 90º
C) 15º
D) 60º
E) 30º
246. Desde un punto exterior a una circunferencia se traza una secante de 16 cm. que determina
una cuerda de 5 cm. Si el punto está a 15 cm. del centro de la circunferencia, el radio de ella
mide
A) 5 cm. B) 6,5 cm. C) 7 cm. D) 7,5 cm. E) 8 cm.
247. En una circunferencia y son cuerdas. La intersección de ambas cuerdas es M.
= 12, = 8, = 2. Entonces =
A) 64 B) 32 C) 16 D) 4 E) 1
248. En la figura, el cuadrado tiene lado de medida a, además : = 1 : 4. Entonces la longitud
de es
A)
B)
C)
D)
E)
249. Los catetos de cierto triangulo rectángulo miden 12 y 5 centímetros. ¿Cuál es la medida de la
altura correspondiente a la hipotenusa?
A) B) C) D) E)
Danny Perich C.
www.sectormatematica.cl
250. En la figura, 1 y 2 son secantes, 3 es el lugar geométrico de los puntos que equidistan de 1
y 2. Si P es un punto de la recta 3, se puede afirmar que:
I. Los triángulos ABP y CDP tienen la misma altura.
II. La razón entre las aéreas de los triángulos ABP y CDP es igual a la razón entre la bases
y .
III. El BPC es ángulo recto siempre.
A) solo I
B) solo II
C) solo III
D) solo I y II
E) Solo I y III
251. En el triangulo ABC se tiene que y son transversales de gravedad y se interceptan en
forma perpendicular en G. Si = 3 y = 2, entonces mide:
A) 2
B) 2
C) 2
D) 10
E) 18
252. En una circunferencia de 20 m de diámetro, la distancia desde el centro a una cuerda es
6 m. La cuerda mide:
A) 8 m. B) 10 m. C) 12 m. D) 16 m. E) no es posible de calcular
253. En la figura, es un diámetro de la circunferencia de centro O y radio r.
es tangente en P y mide r. Si M es el punto medio de , entonces la
longitud de , en términos de r, es
A) r B) C) D) E)
254. Se tiene un triangulo equilátero ABC de lado 4. D es un punto de y E es un punto de ,
además y son perpendiculares entre sí, lo mismo que y . Entonces mide
A) 2 B) 3 C) D) 2 E)
255. Una circunferencia de centro O y radio 3 es tangente interiormente a una circunferencia de
diámetro . Si A es el punto de tangencia, O está en , cuya medida es 8 cm. y es una
cuerda de la circunferencia mayor, tangente a la circunferencia menor en C, ¿cuánto mide
?
A) 1,8 cm. B) 2,4 cm. C) 2,5 cm. D) 3,0 cm. E) 3,6 cm.
Danny Perich C.
www.sectormatematica.cl
256. Se tienen dos circunferencias concéntricas, un segmento de medida 2 cm., el punto N
perteneciente a la circunferencia exterior y el punto M a la circunferencia interior. Si es
tangente a la circunferencia interior, ¿cuál es el área del anillo?
A) 2cm2
B) 4cm2
C) 2 cm2
D) 4 cm2
E) no se puede determinar.
257. En una circunferencia, los segmentos de una de dos cuerdas que se interceptan miden 8 y 9
cm respectivamente. Sabiendo que uno mide el doble del otro, las medidas del los segmentos
de la otra cuerda son
A) 5 cm y 10 cm B) 7 cm y 10 cm C) 8 cm y 16 cm D) 6 cm y 12 cm E) 7 cm y 14 cm
258. Sea ABC un triangulo equilátero de lado a, M es un punto del arco BC en la circunferencia
circunscrita, como indica la figura. Si b ¿cuánto mide ?
A) a – b
B)
C)
D)
E) no se puede determinar
259. En una circunferencia , y son cuerdas, corta a en P y a en Q.
=9, = 4, = 10, = 2 y = 6. Entonces mide
A) 4 B) 6 C) 10 D) 12 E) 20
260. En la circunferencia de centro O y diámetro de la figura,  . Si = 8 cm y
= 4 cm, ¿cuánto mide el perímetro de la circunferencia?
A) 16cm
B) 20cm
C) 32cm
D) 80cm
E) 100cm
261. Si el radio de un cono se duplica, entonces su volumen
A) se mantiene igual B) se duplica C) se reduce a la mitad
D) se cuadruplica E) se reduce a la cuarta parte
262. El triangulo ABC es rectángulo en C. es transversal de gravedad sobre . ¿Cuánto mide
el ángulo b?
A) 20o
B) 110o
C) 55o
D) 70o
E) 32,5o
Danny Perich C.
www.sectormatematica.cl
263. El área de un hexágono regular es de 24 luego de aplicar una homotecia se obtiene un
hexágono regular de área 6 . ¿Cuál es el factor de homotecia?
A) 0,5 B) 0,25 C) 2 D) 4 E) 1,5
264. Los lados de un rectángulo están en la razón 3 : 2. Si el lado mayor mide 3 unidades más que
el lado menor, ¿cuánto mide el área del rectángulo?
A) 108 cm2
B) 96 cm2
C) 54 cm2
D) 40 cm2
E) 24 cm2
265. En el triangulo ABC se tiene que es la bisectriz de ángulo BAC. =12 cm.;
= 2 cm.; = 14 cm., el valor del segmento s es
A) 5,5 cm.
B) 10,5 cm.
C) 12 cm.
D) 6 cm.
E) ninguna de las anteriores.
266. En la figura, dadas las dimensiones del rectángulo ABCD, entonces la medida del lado
en el rectángulo DBEF mide
A)
B)
C)
D)
E) 1
267. Considere un paralelogramo ABCD. Siendo M el punto medio del lado y O el punto de
intersección del trazo con la diagonal ; entonces se tiene que =
A) B) C) D) E)
268. Se tiene un cuadrado AFED y un triangulo BAC rectángulo en A, tal que, D está entre B y A;
F está entre A y C; E está entre B y C; = 1 y = 3. ¿Cuánto mide el lado del cuadrado?
A) 0,70 B) 0,75 C) 0,80 D) 0,85 E) 0,90
269. Un punto Q divide en sección áurea a un trazo , con > . Si = 10 cm. y
= x, entonces la ecuación para determinar x es
A) x2
+ 10x – 100 = 0 B) x2
- 10x + 100 = 0 C) x2
- 10x – 100 = 0
D) x2
+ 10x + 100 = 0 E) x2
+ x – 100 = 0
Danny Perich C.
www.sectormatematica.cl
270. Dada una circunferencia de centro O y radio a, con ⊥ y = b, el valor = x está
dado por la ecuación
A) x2
- 2ax – b2
= 0
B) x2
- 2ax + b2
= 0
C) x2
+ 2ax – b2
= 0
D) x2
+ 2ax + b2
= 0
E) x2
- ax + b2
= 0
271. En la figura = 6,5; = 12, O centro de la semi-circunferencia. El área del triangulo
ABC es
A) 20
B) 30
C) 40
D) 60
E) 120
272. // entonces la medida de es
A) 15
B) 16
C) 25
D) 32
E) otro valor
273. La cuerda es diámetro y la simetral de la cuerda ; P es
punto de ; EP se extiende hasta cortar la circunferencia en A;
entonces, el triangulo EPM es semejante con el triangulo
A) EFA
B) EFC
C) ABM
D) ABP
E) FMC
274. En la figura = 12; = 7; = 9, entonces mide
A) 12
B) 10,5
C) 7,5
D) 4
E) 6
275. La razón entre la altura del triangulo ABC y la altura correspondiente del triangulo MNP es
7:10. Si los triángulos son semejantes, ¿cuál es la razón entre las aéreas de estos triángulos?
A) 7 : 10 B) 10 : 7 C) 49 : 100 D) 343 : 1000 E) 1000 : 343
Danny Perich C.
www.sectormatematica.cl
276. Dos octógonos regulares tienen lados cuyas longitudes son 6 y 9. La razón de sus aéreas
es
A) 4 : 3 B) 2 : 3 C) 2 : D) : 9 E) 2 : 1
277. Se tiene un segmento de 95 cm. Al dividirlo interiormente por dos puntos dados P y Q
tales que : : = 3 : 5 : 11, la diferencia entre el mayor y el menor de los segmentos
que resultan de tal división es
A) 15 cm. B) 25 cm. C) 40 cm. D) 55 cm. E) 60 cm.
278. Las diagonales de un rombo miden 30 y 40. ¿Cuánto mide el radio de la circunferencia
inscrita en él?
A) 12 B) 15 C) 18 D) 21 E) 25
279. En el triangulo ABC de la figura adjunta, // . Además = a, = 1,4a y el área del
triangulo ABC es 98 cm. El área del triangulo EFC es
A) 35 cm.
B) 42 cm.
C) 49 cm.
D) 50 cm.
E) 56 cm.
280. Si MNPQ es un rectángulo y ⊥ , entonces, de acuerdo a los datos de la figura, es
equivalente a
A) B) C)
D) E)
281. ¿Cuál(es) de las siguientes afirmaciones es (son) verdadera(s)?
I) El rombo tiene ejes de simetría y centro de simetría.
II) El romboide tiene centro de simetría, pero no tiene ejes de simetría.
III) El trapecio isósceles tiene eje de simetría, pero no tiene centro de simetría.
A) Solo I B) Solo I y II C) Solo I y III D) I, II y III E) Ninguna de ellas
282. El perímetro de un rectángulo es de 46 cm. Si el largo disminuye en 3 cm. y el ancho
aumenta en 2 cm., el área del rectángulo no cambia. En estas condiciones, la diferencia de
las medidas originales entre el largo y el ancho es
A) 15 cm. B) 12 cm. C) 8 cm. D) 7 cm. E) 5 cm.
Danny Perich C.
www.sectormatematica.cl
283. Los lados de un triángulo miden 2 , 6 y 8 , respectivamente. ¿Cuál es la longitud de la
menor de las alturas de este triángulo?
A) 2 B) 6 C) 8 D)
2
2
E)
2
6
284. es altura del triángulo rectángulo ABC. Si : = 1 : 2 y = 4, ¿cuál(es) de las
siguientes relaciones es(son) verdadera(s)?
I) 2 >
II) <
III) 2 >
A) Sólo II B) Sólo I y II C) Sólo I y III
D) Sólo II y III E) I, II y III
285. ABCD es un cuadrado de lado a, en que lo divide en un triángulo y en un trapecio cuyas
áreas están en la razón 1 : 4. ¿Cuál es el valor de ?
A)
4
1
a B)
5
2
a C)
3
1
a
D)
4
3
a E)
5
3
a
286. En la figura se tiene un círculo de centro O y otros tres semicírculos de radio 6 y centros O,
O1
y O2
. ¿Cuál es el área de la región achurada?
A) 3 – 3 B) 5 + 3 C) 7 – 3
3
D) 4 + 3
3 E) 8 – 3
287. La circunferencia más grande de la figura tiene radio R y las otras dos tienen radio r. Si
10

 r
R y E es centro de la circunferencia, entonces + =
A) 95
B) 96
C) 97
D) 98
E) 100
288. La suma de dos ángulos exteriores de un triángulo es igual a 270°. Si el lado mayor mide
48 cm., ¿cuánto mide el trazo que une el circuncentro con el centro de gravedad?
A) 6 cm. B) 8 cm. C) 12 cm. D) 16 cm. E) 20 cm.
Danny Perich C.
www.sectormatematica.cl
289. En la figura, ΔDFE es rectángulo en F, ADCB es trapecio
isósceles de lados = = 5, = 4 y = 10. Si B es
punto medio de , entonces =
A) 3,25
B) 3,5
C) 3,75
D) 4,0
E) 4,25
290. Si en un rombo de área k, la diagonal mayor es el doble de la menor, entonces el lado del
rombo en términos de k es
A) k B) k
2
2
1
C) k
3
3
1
D) k
4
4
1
E) k
5
2
1
291. En la figura, P divide a en la razón 2 : 3 y Q divide a en la razón 3 : 4. Si = 2 cm,
entonces la longitud de es
A) 60 cm.
B) 70 cm.
C) 75 cm.
D) 80 cm.
E) 85 cm.
292. En la figura, se han dibujado 3 semicírculos, tangentes entre sí, con diámetros , y y.
Si  , entonces la razón entre el área de la región achurada y el área del círculo de
diámetro es
A) 1 : 2
B) 1 : 3
C) 3 : 7
D) 1 : 1
E) 2 : 6
293. Un círculo de radio r está contenido en otro de radio R. Si el área del círculo mayor es
b
a
veces el área de la región comprendida entre el círculo menor y el mayor, entonces
r
R
es
igual a
A)
b
a
B)
b
a
a

C)
b
a
b

D)
b
a
a

E)
b
a
b

294. En el triángulo ABC de la figura, se traza de manera tal que : = 1 : 2, dimidia a
, luego : =
A) 1 : 4
B) 1 : 3
C) 2 : 5
D) 4 : 11
E) 3 : 8
Danny Perich C.
www.sectormatematica.cl
295. En la figura 6, las líneas segmentadas son bisectrices de los ángulos interiores del rectángulo
ABCD y la región achurada es un cuadrado. Si los lados del rectángulo son a y b, entonces
para determinar el área del cuadrado se debe conocer
(1) a – b
(2) ab
A) (1) por sí sola
B) (2) por sí sola
C) Ambas juntas, (1) y (2)
D) Cada una por sí sola, (1) ó (2)
E) Se requiere información adicional
296. La figura muestra un cuadrado de lado L en el cual se ha inscrito un rectángulo de lados a y
b. Entonces, L en función de a y b, es igual a
2
b
a
)
A

B) 2
2
b
a  C) ab
D)
2
b
a 
E) ab
2
297. Si en un triángulo isósceles de base 2 cm, el ángulo del vértice mide 45º, entonces la medida
de uno de sus lados iguales es
A) 3 cm. B) 2
2 cm. C) )
2
2
4
(  cm. D) 2
2
4  cm. E) Falta información
298. En la figura, // , = = y ABC = 34°, ¿cuánto mide el AEC?
A) 34°
B) 51°
C) 67°
D) 68°
E) 90°
299. En el rectángulo ABCD de la figura, P es un punto interior tal que = 3 cm, = 4 cm y
= 5 cm, ¿cuánto mide ?
A) 3
2 cm.
B) 2
3 cm.
C) 3
3 cm.
D) 2
4 cm.
E) 2 cm.
Danny Perich C.
www.sectormatematica.cl
300. En el lado del triángulo ABC se tiene un punto E de forma que : = 1 : 3 y en el lado
un punto D tal que : = 1 : 2. Si F es el punto de intersección de y , entonces
FD
AF
FC
EF
 es igual a
A)
5
4
B)
4
5
C)
2
3
D) 2 E)
2
5
301. ABCD es un cuadrado de lados 1 cm. En los lados y se toman, respectivamente, los
puntos E y F, de modo que = . ¿Cuál es el área máxima del cuadrilátero CDFE?
A)
2
1
cm2
B)
16
9
cm2
C)
32
19
cm2
D)
8
5
cm2
E)
3
2
cm2
302. En la figura, el triángulo PQR es equilátero y está formado por cuatro triángulos equiláteros.
Si se hace girar la figura en torno a la altura del ΔPQR, entonces el volumen del sólido
generado por el triángulo achurado de lado L es
A)
B)
C)
D)
E)
303. Una esfera de volumen V se calienta hasta que su radio se incrementa en un 10%. ¿Cuál es
el nuevo volumen de la esfera?
A) 10
-3
V B) 1,1 V C) 1,21 V D) 1,030 V E) 1,331 V
304. En la figura, el área del triángulo OAD es igual a del área del trapecio isósceles OCBA.
¿Cuáles son las coordenadas del punto medio de ?
A) (5, 5)
B) (4, 5)
C) (5, 6)
D) (4, 3)
E) (5, 4)
Danny Perich C.
www.sectormatematica.cl
305. En cierto triángulo ABC se dibuja la transversal de gravedad . En el triángulo DBC se dibuja
la transversal de gravedad , que mide 9 cm. Sobre se toma un punto F de modo que
// . ¿Cuánto mide ?
A) 3 cm B) 4,5 cm C) 5 cm D) 6 cm E) 6,2 cm
306. En la figura, ΔABC es isósceles de base = b y altura = h. ¿Cuál es el área del rectángulo
DEHG si = x?
A)
B)
C)
D) x(b – x)
E) x(h – x)
307. El área de la figura es de 720 mm
2
. Si la figura está formada por un cuadrado, por dos
trapecios isósceles congruentes y dos triángulos congruentes, todos de igual área, entonces
¿cuánto mide la distancia x?
A) 16 mm
B) 18 mm
C) 20 mm
D) 22 mm
E) 24 mm
308. Se tiene el triángulo ABC isósceles rectángulo en A. Sus catetos
miden 1. Si DF
y
DE
,
AD son radios de la semicircunferencia y DF
es perpendicular a BC . ¿Cuánto vale el radio de la
semicircunferencia inscrita?
A) B) C) D) E)
309. El triangulo ABC de la figura es isósceles de base BC, si BD = BE y
CB = CD, entonces ¿cuál(es) de la(s) siguientes afirmaciones es(son) verdadera(s)?
I) BAC = DCB = EBD
II) ∆ADC  ∆ CEB
III) ∆BEC es isósceles
A) Sólo I
B) Sólo II
C) Sólo I y II
D) Sólo I y III
E) Sólo II y III
Danny Perich C.
www.sectormatematica.cl
310. La hipotenusa de un triángulo rectángulo mide 10 cm y el radio de la circunferencia inscrita
1 cm. El perímetro del triángulo es
A) 15 cm B) 22 cm C) 24 cm D) 26 cm E) 30 cm
311. La altura que llega a la base de un triángulo isósceles de perímetro 32, mide 8, el área del
triángulo es
A) 56 B) 48 C) 40 D) 32 E) 24
312. En la figura, ABCD es cuadrado, M es punto medio de y N
es punto medio de . El cociente entre el área de AOCD y el
área de ABCD es
A) B) C)
D) E)
313. El volumen engendrado al rotar un rombo de diagonales 6 y 8, por uno de sus lados, es
A) B) C) D) E) Otro valor
314. En la figura, polígono de vértices A, B y C, cuyas
coordenadas son: (-1, -2); (2, -2) y (2, 2), respectivamente.
Si se le aplica una rotación de 90° con centro en A, ¿cuál
será la coordenada del vértice C del polígono en la nueva
posición?
A) (1, -5) B) (-5, 1) C) (3, -5)
D) (-5, 3) E) (3, 5)
315. Las coordenadas del punto (x, y), perteneciente al segundo cuadrante, después de una
simetría central con respecto al origen del sistema cartesiano está representado por
A) (x, y) B) (x, -y) C) (-x, y) D) (-x, -y) E)
x y
,
2 2
 
 
 
316. En la circunferencia de centro O de la figura, ¿cuál(es) de las siguientes afirmaciones es
(son) verdadera(s)?
I) ∙ = ∙
II) = ∙
III) ∙ = ∙
A) Solo I B) Solo II C) Solo I y III
D) Solo II y III E) I, II y III
Danny Perich C.
www.sectormatematica.cl
317. Se tiene un triángulo ABC equilátero de altura . Se realiza una homotecia con centro C y
razón = -2, el área del triángulo homotético A’B’C’ es
A) 9 B) 12 C) 36 D) 42 E) 45
318. En la figura, se tienen dos circunferencias congruentes y tangentes exteriores de radio 6 cm.
Si AB contiene los centros de las circunferencias y BT es tangente en T, entonces la medida
de la cuerda BP es
A) 12 2 B) 8 2 C) 6 2
D) 4 2 E) 3 2
319. En la circunferencia de la figura Nº 3, M es el punto medio del arco BMC, ⊥ AB. Si = x y
= x + 1, entonces =
A) 3x + 2
B) 3x + 1
C) 2x + 3
D) 2x + 2
E) 2x + 1
320. En la figura, ABCDEF es hexágono regular, O es el centro del
hexágono, es perpendicular a y M es la intersección de y
. Si = 8, entonces ¿cuánto mide ?
A) 3
B)
C)
D) 4
E)
A
T
P
B
Danny Perich C.
www.sectormatematica.cl
ALTERNATIVAS CORRECTAS
1 B
2 D
3 A
4 B
5 B
6 C
7 E
8 A
9 D
10 B
11 E
12 D
13 A
14 A
15 D
16 D
17 A
18 B
19 A
20 B
21 E
22 A
23 D
24 C
25 D
26 D
27 A
28 A
29 D
30 A
31 B
32 D
33 B
34 B
35 B
36 C
37 E
38 C
39 E
40 D
41 B
42 A
43 E
44 A
45 E
46 B
47 B
48 A
49 B
50 D
51 A
52 A
53 D
54 C
55 C
56 E
57 C
58 A
59 D
60 E
61 D
62 B
63 E
64 C
65 D
66 E
67 A
68 C
69 E
70 E
71 D
72 D
73 C
74 B
75 B
76 D
77 E
78 B
79 D
80 B
81 E
82 C
83 C
84 E
85 D
86 B
87 C
88 A
89 C
90 D
91 A
92 E
93 E
94 B
95 C
96 D
97 C
98 E
99 D
100 D
101 B
102 D
103 B
104 D
105 E
106 B
107 C
108 B
109 D
110 E
111 C
112 D
113 D
114 A
115 A
116 D
117 B
118 E
119 E
120 A
121 E
122 A
123 B
124 D
125 A
126 C
127 C
128 A
129 D
130 D
131 C
132 A
133 A
134 B
135 E
136 E
137 C
138 B
139 B
140 B
141 D
142 C
143 B
144 A
145 C
146 C
147 A
148 C
149 D
150 C
151 E
152 C
153 D
154 B
155 C
156 A
157 B
158 E
159 B
160 B
161 B
162 E
163 A
164 A
165 A
166 C
167 A
168 E
169 D
170 A
171 B
172 B
173 E
174 B
175 B
176 B
177 C
178 E
179 C
180 A
181 C
182 B
183 D
184 E
185 C
186 B
187 C
188 D
189 E
190 A
191 C
192 A
193 E
194 C
195 D
196 A
197 A
198 C
199 C
200 A
201 B
202 C
203 E
204 E
205 E
206 D
207 E
208 B
209 B
210 C
211 C
212 D
213 B
214 E
215 A
216 E
217 A
218 C
219 E
220 D
221 A
222 B
223 B
224 A
225 E
226 D
227 E
228 D
229 C
230 E
231 A
232 A
233 A
234 D
235 B
236 C
237 E
238 B
239 A
240 E
241 A
242 B
243 E
244 D
245 E
246 C
247 C
248 A
249 D
250 D
251 D
252 D
253 B
254 B
255 B
256 B
257 D
258 C
259 A
260 B
261 D
262 E
263 A
264 C
265 B
266 D
267 D
268 B
269 A
270 C
271 B
272 C
273 A
274 E
275 C
276 A
277 C
278 A
279 D
280 D
281 B
282 D
283 E
284 E
285 B
286 C
287 E
288 B
289 D
290 E
291 B
292 D
293 B
294 B
295 A
296 A
297 D
298 B
299 B
300 C
301 D
302 D
303 E
304 E
305 D
306 C
307 C
308 C
309 C
310 B
311 B
312 C
313 B
314 B
315 D
316 A
317 C
318 B
319 E
320 C
Danny Perich C.
www.sectormatematica.cl

Más contenido relacionado

Similar a Geometria (20)

Actividad 4 trigonometria plano cartesiano
Actividad 4 trigonometria  plano cartesianoActividad 4 trigonometria  plano cartesiano
Actividad 4 trigonometria plano cartesiano
 
Guía n°5
Guía n°5Guía n°5
Guía n°5
 
Rectas.pdf
Rectas.pdfRectas.pdf
Rectas.pdf
 
26 ejercicios congruencia de triángulos
26 ejercicios congruencia de triángulos26 ejercicios congruencia de triángulos
26 ejercicios congruencia de triángulos
 
Ma 18 2007
Ma 18 2007Ma 18 2007
Ma 18 2007
 
Ma 18 2007
Ma 18 2007Ma 18 2007
Ma 18 2007
 
38 ecuación de la recta
38 ecuación de la recta38 ecuación de la recta
38 ecuación de la recta
 
Guia 6 transformaciones isometricas
Guia 6 transformaciones isometricasGuia 6 transformaciones isometricas
Guia 6 transformaciones isometricas
 
1 semana trigo
1 semana   trigo1 semana   trigo
1 semana trigo
 
02 transformaciones isometricas
02 transformaciones isometricas02 transformaciones isometricas
02 transformaciones isometricas
 
transformaciones isometricas
transformaciones isometricastransformaciones isometricas
transformaciones isometricas
 
Ma 35 2007
Ma 35 2007Ma 35 2007
Ma 35 2007
 
Ma 35 2007
Ma 35 2007Ma 35 2007
Ma 35 2007
 
Ma 37 2007
Ma 37 2007Ma 37 2007
Ma 37 2007
 
Ma 37 2007
Ma 37 2007Ma 37 2007
Ma 37 2007
 
44 guía de ejercitación-
44  guía de ejercitación-44  guía de ejercitación-
44 guía de ejercitación-
 
áLgebray funciones ecuacióndelarecta
áLgebray funciones ecuacióndelarectaáLgebray funciones ecuacióndelarecta
áLgebray funciones ecuacióndelarecta
 
Rectas en el plano y conicas
Rectas en el plano y conicasRectas en el plano y conicas
Rectas en el plano y conicas
 
G1
G1G1
G1
 
Actividad 5 trigonometria 4to plano cartesiano
Actividad 5 trigonometria 4to plano cartesianoActividad 5 trigonometria 4to plano cartesiano
Actividad 5 trigonometria 4to plano cartesiano
 

Último

Hernandez_Hernandez_Practica web de la sesion 12.pptx
Hernandez_Hernandez_Practica web de la sesion 12.pptxHernandez_Hernandez_Practica web de la sesion 12.pptx
Hernandez_Hernandez_Practica web de la sesion 12.pptxJOSEMANUELHERNANDEZH11
 
PARTES DE UN OSCILOSCOPIO ANALOGICO .pdf
PARTES DE UN OSCILOSCOPIO ANALOGICO .pdfPARTES DE UN OSCILOSCOPIO ANALOGICO .pdf
PARTES DE UN OSCILOSCOPIO ANALOGICO .pdfSergioMendoza354770
 
Plan de aula informatica segundo periodo.docx
Plan de aula informatica segundo periodo.docxPlan de aula informatica segundo periodo.docx
Plan de aula informatica segundo periodo.docxpabonheidy28
 
International Women's Day Sucre 2024 (IWD)
International Women's Day Sucre 2024 (IWD)International Women's Day Sucre 2024 (IWD)
International Women's Day Sucre 2024 (IWD)GDGSucre
 
Proyecto integrador. Las TIC en la sociedad S4.pptx
Proyecto integrador. Las TIC en la sociedad S4.pptxProyecto integrador. Las TIC en la sociedad S4.pptx
Proyecto integrador. Las TIC en la sociedad S4.pptx241521559
 
Presentación inteligencia artificial en la actualidad
Presentación inteligencia artificial en la actualidadPresentación inteligencia artificial en la actualidad
Presentación inteligencia artificial en la actualidadMiguelAngelVillanuev48
 
CLASE DE TECNOLOGIA E INFORMATICA PRIMARIA
CLASE  DE TECNOLOGIA E INFORMATICA PRIMARIACLASE  DE TECNOLOGIA E INFORMATICA PRIMARIA
CLASE DE TECNOLOGIA E INFORMATICA PRIMARIAWilbisVega
 
SalmorejoTech 2024 - Spring Boot <3 Testcontainers
SalmorejoTech 2024 - Spring Boot <3 TestcontainersSalmorejoTech 2024 - Spring Boot <3 Testcontainers
SalmorejoTech 2024 - Spring Boot <3 TestcontainersIván López Martín
 
La era de la educación digital y sus desafios
La era de la educación digital y sus desafiosLa era de la educación digital y sus desafios
La era de la educación digital y sus desafiosFundación YOD YOD
 
Cortes-24-de-abril-Tungurahua-3 año 2024
Cortes-24-de-abril-Tungurahua-3 año 2024Cortes-24-de-abril-Tungurahua-3 año 2024
Cortes-24-de-abril-Tungurahua-3 año 2024GiovanniJavierHidalg
 
Medidas de formas, coeficiente de asimetría y coeficiente de curtosis.pptx
Medidas de formas, coeficiente de asimetría y coeficiente de curtosis.pptxMedidas de formas, coeficiente de asimetría y coeficiente de curtosis.pptx
Medidas de formas, coeficiente de asimetría y coeficiente de curtosis.pptxaylincamaho
 
Redes direccionamiento y subredes ipv4 2024 .pdf
Redes direccionamiento y subredes ipv4 2024 .pdfRedes direccionamiento y subredes ipv4 2024 .pdf
Redes direccionamiento y subredes ipv4 2024 .pdfsoporteupcology
 
Instrumentación Hoy_ INTERPRETAR EL DIAGRAMA UNIFILAR GENERAL DE UNA PLANTA I...
Instrumentación Hoy_ INTERPRETAR EL DIAGRAMA UNIFILAR GENERAL DE UNA PLANTA I...Instrumentación Hoy_ INTERPRETAR EL DIAGRAMA UNIFILAR GENERAL DE UNA PLANTA I...
Instrumentación Hoy_ INTERPRETAR EL DIAGRAMA UNIFILAR GENERAL DE UNA PLANTA I...AlanCedillo9
 
Global Azure Lima 2024 - Integración de Datos con Microsoft Fabric
Global Azure Lima 2024 - Integración de Datos con Microsoft FabricGlobal Azure Lima 2024 - Integración de Datos con Microsoft Fabric
Global Azure Lima 2024 - Integración de Datos con Microsoft FabricKeyla Dolores Méndez
 
ejercicios pseint para aprogramacion sof
ejercicios pseint para aprogramacion sofejercicios pseint para aprogramacion sof
ejercicios pseint para aprogramacion sofJuancarlosHuertasNio1
 
KELA Presentacion Costa Rica 2024 - evento Protégeles
KELA Presentacion Costa Rica 2024 - evento ProtégelesKELA Presentacion Costa Rica 2024 - evento Protégeles
KELA Presentacion Costa Rica 2024 - evento ProtégelesFundación YOD YOD
 
ATAJOS DE WINDOWS. Los diferentes atajos para utilizar en windows y ser más e...
ATAJOS DE WINDOWS. Los diferentes atajos para utilizar en windows y ser más e...ATAJOS DE WINDOWS. Los diferentes atajos para utilizar en windows y ser más e...
ATAJOS DE WINDOWS. Los diferentes atajos para utilizar en windows y ser más e...FacuMeza2
 
trabajotecologiaisabella-240424003133-8f126965.pdf
trabajotecologiaisabella-240424003133-8f126965.pdftrabajotecologiaisabella-240424003133-8f126965.pdf
trabajotecologiaisabella-240424003133-8f126965.pdfIsabellaMontaomurill
 
guía de registro de slideshare por Brayan Joseph
guía de registro de slideshare por Brayan Josephguía de registro de slideshare por Brayan Joseph
guía de registro de slideshare por Brayan JosephBRAYANJOSEPHPEREZGOM
 
El gusano informático Morris (1988) - Julio Ardita (1995) - Citizenfour (2014...
El gusano informático Morris (1988) - Julio Ardita (1995) - Citizenfour (2014...El gusano informático Morris (1988) - Julio Ardita (1995) - Citizenfour (2014...
El gusano informático Morris (1988) - Julio Ardita (1995) - Citizenfour (2014...JaquelineJuarez15
 

Último (20)

Hernandez_Hernandez_Practica web de la sesion 12.pptx
Hernandez_Hernandez_Practica web de la sesion 12.pptxHernandez_Hernandez_Practica web de la sesion 12.pptx
Hernandez_Hernandez_Practica web de la sesion 12.pptx
 
PARTES DE UN OSCILOSCOPIO ANALOGICO .pdf
PARTES DE UN OSCILOSCOPIO ANALOGICO .pdfPARTES DE UN OSCILOSCOPIO ANALOGICO .pdf
PARTES DE UN OSCILOSCOPIO ANALOGICO .pdf
 
Plan de aula informatica segundo periodo.docx
Plan de aula informatica segundo periodo.docxPlan de aula informatica segundo periodo.docx
Plan de aula informatica segundo periodo.docx
 
International Women's Day Sucre 2024 (IWD)
International Women's Day Sucre 2024 (IWD)International Women's Day Sucre 2024 (IWD)
International Women's Day Sucre 2024 (IWD)
 
Proyecto integrador. Las TIC en la sociedad S4.pptx
Proyecto integrador. Las TIC en la sociedad S4.pptxProyecto integrador. Las TIC en la sociedad S4.pptx
Proyecto integrador. Las TIC en la sociedad S4.pptx
 
Presentación inteligencia artificial en la actualidad
Presentación inteligencia artificial en la actualidadPresentación inteligencia artificial en la actualidad
Presentación inteligencia artificial en la actualidad
 
CLASE DE TECNOLOGIA E INFORMATICA PRIMARIA
CLASE  DE TECNOLOGIA E INFORMATICA PRIMARIACLASE  DE TECNOLOGIA E INFORMATICA PRIMARIA
CLASE DE TECNOLOGIA E INFORMATICA PRIMARIA
 
SalmorejoTech 2024 - Spring Boot <3 Testcontainers
SalmorejoTech 2024 - Spring Boot <3 TestcontainersSalmorejoTech 2024 - Spring Boot <3 Testcontainers
SalmorejoTech 2024 - Spring Boot <3 Testcontainers
 
La era de la educación digital y sus desafios
La era de la educación digital y sus desafiosLa era de la educación digital y sus desafios
La era de la educación digital y sus desafios
 
Cortes-24-de-abril-Tungurahua-3 año 2024
Cortes-24-de-abril-Tungurahua-3 año 2024Cortes-24-de-abril-Tungurahua-3 año 2024
Cortes-24-de-abril-Tungurahua-3 año 2024
 
Medidas de formas, coeficiente de asimetría y coeficiente de curtosis.pptx
Medidas de formas, coeficiente de asimetría y coeficiente de curtosis.pptxMedidas de formas, coeficiente de asimetría y coeficiente de curtosis.pptx
Medidas de formas, coeficiente de asimetría y coeficiente de curtosis.pptx
 
Redes direccionamiento y subredes ipv4 2024 .pdf
Redes direccionamiento y subredes ipv4 2024 .pdfRedes direccionamiento y subredes ipv4 2024 .pdf
Redes direccionamiento y subredes ipv4 2024 .pdf
 
Instrumentación Hoy_ INTERPRETAR EL DIAGRAMA UNIFILAR GENERAL DE UNA PLANTA I...
Instrumentación Hoy_ INTERPRETAR EL DIAGRAMA UNIFILAR GENERAL DE UNA PLANTA I...Instrumentación Hoy_ INTERPRETAR EL DIAGRAMA UNIFILAR GENERAL DE UNA PLANTA I...
Instrumentación Hoy_ INTERPRETAR EL DIAGRAMA UNIFILAR GENERAL DE UNA PLANTA I...
 
Global Azure Lima 2024 - Integración de Datos con Microsoft Fabric
Global Azure Lima 2024 - Integración de Datos con Microsoft FabricGlobal Azure Lima 2024 - Integración de Datos con Microsoft Fabric
Global Azure Lima 2024 - Integración de Datos con Microsoft Fabric
 
ejercicios pseint para aprogramacion sof
ejercicios pseint para aprogramacion sofejercicios pseint para aprogramacion sof
ejercicios pseint para aprogramacion sof
 
KELA Presentacion Costa Rica 2024 - evento Protégeles
KELA Presentacion Costa Rica 2024 - evento ProtégelesKELA Presentacion Costa Rica 2024 - evento Protégeles
KELA Presentacion Costa Rica 2024 - evento Protégeles
 
ATAJOS DE WINDOWS. Los diferentes atajos para utilizar en windows y ser más e...
ATAJOS DE WINDOWS. Los diferentes atajos para utilizar en windows y ser más e...ATAJOS DE WINDOWS. Los diferentes atajos para utilizar en windows y ser más e...
ATAJOS DE WINDOWS. Los diferentes atajos para utilizar en windows y ser más e...
 
trabajotecologiaisabella-240424003133-8f126965.pdf
trabajotecologiaisabella-240424003133-8f126965.pdftrabajotecologiaisabella-240424003133-8f126965.pdf
trabajotecologiaisabella-240424003133-8f126965.pdf
 
guía de registro de slideshare por Brayan Joseph
guía de registro de slideshare por Brayan Josephguía de registro de slideshare por Brayan Joseph
guía de registro de slideshare por Brayan Joseph
 
El gusano informático Morris (1988) - Julio Ardita (1995) - Citizenfour (2014...
El gusano informático Morris (1988) - Julio Ardita (1995) - Citizenfour (2014...El gusano informático Morris (1988) - Julio Ardita (1995) - Citizenfour (2014...
El gusano informático Morris (1988) - Julio Ardita (1995) - Citizenfour (2014...
 

Geometria

  • 1. Danny Perich C. www.sectormatematica.cl PSU MATEMATICA: GEOMETRÍA Creación y recopilación de ejercicios de geometría, elaborado con el objetivo de ayudar a los estudiantes a preparar de manera óptima la Prueba de Selección Universitaria (PSU) de matemática. Este trabajo no persigue fines de lucro, por lo consiguiente, este texto se distribuye en forma gratuita. ¡¡Éxito‼ 2017 DANNY PERICH CAMPANA www.sectormatematica.cl
  • 3. Danny Perich C. www.sectormatematica.cl 1. Las coordenadas del punto simétrico de P(-1, 4) respecto al eje de las abscisas es A) (1, -4) B) (-1, -4) C) (1, 4) D) (4, -1) E) (4, 1) 2. La(s) transformación(es) que permite(n) obtener la figura 2, a partir de la figura 1 es A) Rotación B) Traslación C) Simetría Central D) Simetría Axial E) Traslación y rotación 3. Al trasladar el punto (-4, 2), se obtiene el punto (0, 0), entonces el vector de traslación es A) (4, -2) B) (4, 2) C) (-4, -2) D) (2, -4) E) (-2, 4) 4. Se forma un segmento con las coordenadas P(1,3) y Q (-5,2). Al aplicársele una traslación, el nuevo segmento tiene como coordenada imagen de P a P’(-2, 1), entonces la coordenada imagen de Q es A) (-3, -2) B) (-8, 0) C) (-4, -1) D) (2, -5) E) (5, -2) 5. ¿Cuál(es) de las siguientes figuras tiene(n) eje de simetría? I. II. III. A) Sólo I B) Sólo II C) Sólo I y II D) I, II y III E) Ninguna 6. Se efectúa una rotación de 90º al punto (-5, 4), en torno al origen. Las nuevas coordenadas de este punto son A) (-5, -4) B) (5, -4) C) (-4, -5) D) (5, 4) E) (-4, 5) 7. ¿Cuántos ejes de simetría tiene la figura siguiente? A) 1 B) 2 C) 3 D) 4 E) 5 8. Al trasladar el punto A(5, -3) según el vector (-5, 3), la nueva coordenada de A es A) (0, 0) B) (0, 6) C) (0, -6) D) (10, 0) E) (-10, 0) Figura 1 Figura 2
  • 4. Danny Perich C. www.sectormatematica.cl 9. El triángulo ABC de coordenadas A(4, 1), B(2, 5) y C(1, 3) se traslada formando el triángulo de coordenadas A’(7, 3), B’(5, 7) y C’(4, 5). El vector correspondiente a esta traslación es A) (-3, -2) B) (-3, 2) C) (3, -2) D) (3, 2) E) (2, 3) 10. Al punto de coordenada (2, 0) se le aplica una rotación de 90º en torno al punto (2, 2). ¿Cuál es la nueva coordenada de este punto? A) (-2, 0) B) (0, 2) C) (0, -2) D) (2, 2) E) (-2, -2) 11. ¿Cuál(es) de las siguientes afirmaciones es(son) verdadera(s) respecto de una reflexión? I. El perímetro se mantiene inalterable. II. El área no varía. III. Es lo mismo que una simetría. A) Sólo III B) Sólo I y II C) Sólo I y III D) Sólo II y III E) I, II y III 12. Se traslada el segmento AB de coordenadas A(-1, 2) y B(2, 4), hasta el segmento ' B ' A de coordenadas A’(-6, -2) y B’(-3, 0). El vector de traslación es A) (4, 5) B) (4, 3) C) (5, 4) D) (-5, -4) E) (-4, -5) 13. El punto simétrico de P(4, 3) respecto al punto (2, -1) es A) ) 5 , 0 (  B) (6, 2) C) (0, 3) D) (-4, -3) E) (-5, 0) 14. ¿Cuál(es) de las siguientes figuras geométricas tiene(n) más de 2 ejes de simetría? I) El rectángulo II) El cuadrado III) El rombo A) Sólo II B) Sólo I y II C) Sólo II y III D) I, II y III E) Ninguna 15. El punto simétrico a P(-3, 2), con respecto al eje de las ordenadas, es A) (-3, -2) B) (3, -2) C) (-3, 2) D) (3, 2) E) (2, -3) 16. El total de ejes de simetría que tiene un pentágono regular son A) 2 B) 3 C) 4 D) 5 E) 6 17. Los vértices de un triángulo son A(0, 0), B(4, 0) y C(1, 6). Al rotar el triángulo en 270º con respecto al origen, las coordenadas de C resultan A) (6, -1) B) (-1, 6) C) (6, 1) D) (-1, -6) E) (1, -6) 18. Al trasladar el punto A(4, 6) hasta A’(-2 , 3), el vector de traslación es A) (2, 8) B) (-6, -3) C) ) 2 1 , 2 1 ( D) (-8, 18) E) (-2, 2)
  • 5. Danny Perich C. www.sectormatematica.cl 19. Si el punto P(-2, 7) se traslada a P’(4, -1), según el vector de traslación (x – 1, y + 2). Entonces x + y = A) -3 B) -1 C) 0 D) 1 E) 2 20. Una circunferencia de centro (4, 2) y que contiene al punto (0, 0), rota 90º respecto al origen. El nuevo centro está ubicado en A) (-2, -4) B) (-2, 4) C) (-4, 2) D) (-4, -2) E) (4, -2) 21. ¿Cuál de las siguientes alternativas corresponde a una trasformación del punto (-2, -1) en el punto (-1, 2)? A) una traslación B) una rotación en 90º respecto al origen C) una reflexión D) una rotación en 180º respecto del origen E) una rotación en 270º respecto del origen 22. Trasladar el punto A(4, -3) al punto B(2, -1) y luego al punto C(-3,5), equivale a la traslación única A) (-7, 8) B) (3, 1) C) (1, 2) D) (-1, 4) E) (6, -4) 23. Al rotar el cuadrado de vértices A(1, 1), B(4, 1), C(1, 4) y D(4, 4) en 90º y luego trasladarlo según el vector (1, -1), el vértice que queda sobre el origen es A) D B) C C) B D) A E) Ninguno 24. La reflexión, respecto del origen, del punto medio del trazo cuyos extremos son A(6, 2) y B(-4, 4) es A) (1, 3) B) (1, -3) C) (-1, -3) D) (-1, 3) E) (1, 3) 25. ¿Cuál de las siguientes letras no tiene eje de simetría? A) E B) H C) A D) Z E) O 26. Al punto (6, -4) se le aplica una traslación obteniendo el punto (12, -8). Si al punto (-3, 5) se le aplica la misma traslación, entonces se obtiene el punto A) (-6, 10) B) (-9, 9) C) (9, -3) D) (3, 1) E) (6, 9) 27. ¿Cuántos ejes de simetría tiene el romboide? A) 0 B) 1 C) 2 D) 3 E) 4 28. ¿Cuál de las siguientes figuras no tiene centro de simetría? A) Triángulo rectángulo B) Cuadrado C) Rectángulo D) Rombo E) Circunferencia
  • 6. Danny Perich C. www.sectormatematica.cl 29. Al trasladar el triángulo de vértices A(-1,5), B(2,0) y C(3,1), según el vector de traslación (4,1), el vértice homólogo de B es A) (3,6) B) (2,1) C) (6,0) D) (6,1) E) (7,2) 30. Una circunferencia tiene como centro el punto (3,5). Si el vector de traslación de este punto es (-5, 1), ¿Cuál es el centro de la circunferencia trasladada? A) (-2,6) B) (8,6) C) (-2,4) D) (-15,5) E) (8,4) 31. El triángulo que resulta al rotar, con centro en el origen y ángulo de 180º, el triángulo de vértices: A = (2,3), B = (7,-2) y C = (5,8), tiene coordenadas A) A = (2,3), B = (7,-2) y C = (5,8) B) A = (-2,-3), B = (-7,2) y C = (-5,-8) C) A = (3,2), B = (-2,7) y C = (8,5) D) A = (3,-2), B = (-2,-7) y C = (8,-5) E) A = (-2,3), B = (-7,-2) y C = (-5,8) 32. El vector traslación que transforma el punto (2,5) en el punto (-9,2), es A) (11,3) B) (-7,3) C) (-7,-7) D) (-11,-3) E) (11,-3) 33. Si al triángulo ABC, de coordenadas A(2, 2), B(2, -4) y C(6, -1), se le aplica una rotación de 90º, con centro en el origen, y luego una traslación T(5,-2), el vértice C sería A) (1,6) B) (6,4) C) (11,-3) D) (1,1) E) Otro 34. Dados los siguientes triángulos, determinar cuáles son congruentes. I) II) III) A) Sólo I B) Sólo I y II C) Sólo I y III D) Sólo II y III E) I, II y III 35. Un alumno para demostrar en el cuadrado de la figura que ABC  DBC, determinó que  , que  y que el CAB  CDB, por ser rectos. ¿Qué criterio de congruencia utilizó? A) LLL B) LAL C) ALA D) AAL E) LLA 75° 8 cm. 65 ° 75° 8 cm. 65 ° 75° 8 cm. 65 ° C D B A
  • 7. Danny Perich C. www.sectormatematica.cl 36. En la figura siguiente, ABD  CDB y ADB  CBD. ¿Qué criterio de congruencia permite demostrar que el ABD  CDB? A) LLL B) LAL C) ALA D) AAL E) LLA 37. Dos triángulos son congruentes si A) Tienen dos pares de ángulos respectivamente iguales B) Tienen dos pares de lados respectivamente iguales C) Tienen igual área D) Tienen igual perímetro E) Tienen sus lados respectivos en razón 1:1 38. Los lados de un triángulo rectángulo miden 6 cm. y 8 cm. El perímetro de un triángulo congruente al anterior es A) 10 B) 14 C) 24 D) 48 E) No se puede determinar 39. ¿En cuál(es) de las siguientes figuras se determina(n) dos triángulos congruentes al trazar una de sus diagonales? I. Cuadrado II. Rectángulo III. Rombo A) Sólo I B) Sólo I y II C) Sólo I y III D) Sólo II y III E) I, II y III 40. Al trazar las diagonales de un cuadrado es falso que se formen cuatro triángulos A) Congruentes B) Isósceles C) Rectángulos D) Equiláteros E) De igual área 41. En la figura, AC  DF y BC  EF . El valor de x es A) 15º B) 30º C) 60º D) 90º E) 150º 42. En la figura, el CDE es isósceles. C es punto medio de y D es punto medio de . ¿Qué criterio de congruencia permite demostrar que el ACE  BDE? A) LAL B) ALA C) LLA D) LLL E) AAL A D C B E C B A D 30 30 A C B D E F 3x 2x
  • 8. Danny Perich C. www.sectormatematica.cl 43. En los triángulos siguientes se verifica que  , que  y que el CAB  FDE. ¿Qué criterio permite demostrar que estos triángulos son congruentes? A) LLL B) LAL C) ALA D) LLA E) Falta Información 44. En la figura, el ABC  DEF, siendo CAB  FDE y ACB  DFE, entonces se verifica que A) AC  DF B) BC  DE C) AB  FE D) AC  FE E) AB  FD 45. Para demostrar que los triángulos AOB y COD de la figura, son congruentes, es necesario saber que A) AB  DC B) BAO  DCO C) AB // CD D) AO  DO y AB  CD E) BO  CO y AO  DO 46. Los triángulos ABC y DEF de la figura son congruentes, entonces la medida de es A) 9 B) 15 C) 17 D) 40 E) Falta información 47. Marca la alternativa de la proposición verdadera A) Dos triángulos rectángulos son congruentes si sus ángulos agudos respectivos son congruentes. B) Dos triángulos son congruentes si sus lados homólogos miden lo mismo. C) Dos triángulos son congruentes si sus ángulos respectivos son iguales. D) Para demostrar que dos triángulos son congruentes se puede utilizar el criterio AAL. E) Todos los triángulos equiláteros son congruentes. 48. En la figura, ABC equilátero y   . El criterio que permite demostrar que los triángulos AFE, BDF y CED son congruentes es: A) LAL B) LLL C) ALA D) LLA E) LAA A B C D F E C A B D F E B O A D C C E D F B A A C 17 B 40 80 15 F D E 60 80
  • 9. Danny Perich C. www.sectormatematica.cl 49. El área del rombo cuyas diagonales son x – 2 y x + 2 es A) 4 x2  B) 2 4 x2  C) 4 4 x2  D) 8 4 x2  E) No se puede determinar 50. El área de un triángulo equilátero de lado 6 es A) 36 B) 18 C) 2 3 6 D) 3 9 E) Ninguna de las anteriores 51. Si la diagonal de un cuadrado mide 3 cm., entonces el radio de la circunferencia inscrita al cuadrado es A) 4 2 3 cm. B) 1,5 cm. C) 2 2 3 cm. D) 3 cm. E) Ninguna de las anteriores 52. El área de un triángulo equilátero es 3 18 cm2 , ¿cuál es su perímetro? A) 2 18 cm. B) 3 6 cm. C) 3 18 D) 2 6 E) 3 54 53. Las diagonales de un rombo son 10 cm. y 24 cm. ¿Cuánto mide el área del cuadrado construido sobre uno de los lados del rombo? A) 240 cm2 B) 120 cm2 C) 52 cm2 D) 169 cm2 E) 26 cm2 54. Si el lado de un cuadrado aumenta 3 veces, entonces su área aumenta A) 3 veces B) 6 veces C) 9 veces D) 12 veces E) 24 veces 55. Si la arista a de un cubo aumenta en 2 unidades, ¿en cuánto aumenta su área? A) 2 ) 2 a (  B) 2 ) 2 a ( 6  C) ) 1 a ( 24  D) ) 4 a ( 6 2  E) Ninguna de las anteriores 56. La mediana de un trapecio mide 40 cm. Si la altura corresponde a los 4 3 de la mediana, ¿cuál es el área del trapecio? A) 10 cm2 B) 900 cm2 C) 1.600 cm2 D) 600 cm2 E) 1.200 cm2 57. El radio de una circunferencia tiene la misma medida que la diagonal de un cuadrado de área 16 cm2 . El área del círculo es A)  4 cm2 B)  16 cm2 C)  32 cm2 D)  8 cm2 E)  2 4 cm2 58. Los lados de un triángulo equilátero y de un cuadrado están en razón 1 : 2. ¿Cuál es la razón entre sus áreas? A) 3 : 16 B) 3 : 4 C) 1 : 4 D) 1 : 2 E) Ninguna de las anteriores
  • 10. Danny Perich C. www.sectormatematica.cl 59. El lado de un cuadrado mide 4 log2 cm. ¿Cuántos cm2 mide su área? A) 8 log2 B) 4 log4 C) 16 log4 D) 81 log3 E) Ninguna de las anteriores 60. Al rotar un cuadrado de diagonal 4 cm., con respecto a uno de sus lados, se obtiene una figura cuya área es A) 16 cm2 B) 8 cm2 C)  16 cm2 D)  8 cm2 E)  2 16 cm2 61. La diferencia entre el área de un cuadrado de lado 10 m. y el área de un triángulo equilátero de altura 3 10 m es A) 10 – 10 B) 100-10 C) 100-25 D) 100-100 E) Otro valor 62. Los lados de un triángulo son a, 2a y 3a. Entonces su perímetro es A) 5a B) 6a C) 3 a 5 D) 3 a 6 E) Falta Información 63. El área de un rectángulo de lados a y (a + b) es A) 2a + b B) 4a + 2b C) b a2  D) ab a 2  E) ab a2  64. El área de un cuadrado es 4 cm2 . ¿Cuál es el perímetro del triángulo equilátero construido sobre su diagonal? A) 8 cm. B) cm. C) cm. D) cm. E) cm. 65. Con la quinta parte del perímetro de una circunferencia se construye una circunferencia de 16 cm. de longitud. ¿Cuál es el radio de la circunferencia mayor? A) 8 cm. B) 16 cm. C) 20 cm. D) 40 cm. E) 80 cm. 66. El área de un cuadrado es 64 cm2 . Si cada lado disminuye a la cuarta parte, ¿cuánto mide la mitad del área del cuadrado resultante? A) 32 cm2 B) 16 cm2 C) 8 cm2 D) 4 cm2 E) 2 cm2 67. Si en un triángulo equilátero la longitud de cada lado aumenta en una unidad, entonces ¿cuál de las siguientes afirmaciones es verdadera? A) su perímetro aumenta en 3 unidades B) su área aumenta en 3 unidades cuadradas C) su perímetro permanece constante D) su área permanece constante E) su altura aumenta en 1 unidad 68. La superficie de un cuadrado es 4x2 + 4x + 1. Si el lado aumenta en 2 unidades, su área aumenta en A) 2 cm2 B) 4 cm2 C) (8x + 8) cm2 D) 8 cm2 E) 8x cm2
  • 11. Danny Perich C. www.sectormatematica.cl 69. Cada arista del cubo de la figura, mide 2 cm. ¿Cuánto mide la superficie del cuadrilátero sombreado? A) 4 cm2 B) 8 cm2 C) 16 cm2 D) 2 2 cm2 E) 2 4 cm2 70. La superficie de un cubo es 6x2 – 12x + 6, si la arista disminuye en 1 unidad, el área de una de sus caras es A) 2 x B) 5 x 12 x 6 2   C) 1 x 2 x2   D) 4 x2  E) 4 x 4 x2   71. En la circunferencia de centro O de la figura, CAO = 20º, AOB = 100º. El valor del OBC es A) 10º B) 20º C) 25º D) 30º E) 40º 72. En la circunferencia de centro O, AO AB  . La medida del COA es 140º, entonces x mide A) 40º B) 50º C) 90º D) 130º E) 140º 73. En la figura siguiente, OB es radio de la circunferencia. Si CAB = 70º y ABO = 25º, entonces ACB = A) 25º B) 45º C) 65º D) 80º E) 130º 74. Los arcos MN, NP y PQ de la circunferencia de centro O, son de igual medida. Si MOP = 112º, la medida del PRQ es A) 14º B) 28º C) 34º D) 56º E) 68º A B O C A x B O C A B O C R O M P Q N
  • 12. Danny Perich C. www.sectormatematica.cl 75. En la circunferencia de centro O y diámetro AD de la figura, los arcos AB, BC y CD están en la razón 6 : 3 : 1. El valor de x es A) 108º B) 54º C) 30º D) 27º E) 18º 76. AD y CD tangentes a la circunferencia de centro 0. Si BC AB  y BC AC  , entonces ADC mide A) 15º B) 30º C) 45º D) 60º E) 90º 77. En la semicircunferencia de centro O, OBC = 40º. La medida del AOC es A) No se puede determinar B) 20º C) 40º D) 60º E) 80º 78. En la figura, OC // AB , AD diámetro de la circunferencia de centro O. Si el ángulo BAD mide 40º, entonces el BOC mide A) 20º B) 40º C) 60º D) 80º E) 100º 79. En la circunferencia de centro O siguiente, la medida del ángulo x es A) No se puede determinar B) 90º C) 45º D) 60º E) 30º 80. En la circunferencia de centro O, AO AB  . La medida del ACB es A) 15º B) 30º C) 45º D) 60º E) Faltan datos 81. A un cuadrado se le circunscribe una circunferencia de radio r. La razón entre sus áreas, respectivamente es A) 2 : 1 B) 2 :  C) 4 :  D)  : 1 E)  : 2 x C O A B D A C B O D C B O A O B A D C x x O A B C O .
  • 13. Danny Perich C. www.sectormatematica.cl 82. El área de un círculo es  , entonces su perímetro mide A) 2  B)  C)  2 D)  4 E) Ninguna de las anteriores 83. Si el radio de una circunferencia y la diagonal de un cuadrado miden lo mismo, entonces la razón entre sus respectivos perímetros es A) 2 :  B) 2 4 :  C) 2 :  D) 2 : 2 E) 2 2 :  84. El radio de la circunferencia circunscrita del hexágono regular de área 27 es A) 2 3 B) 2 C) 2 2 D) 2 2 E) 2 85. El volumen de un cubo cuya área total es 12 cm2 es A) 2 cm3 B) 8 cm3 C) 2 cm3 D) 2 2 cm3 E) 3 12 cm3 86. El volumen, en cm3 , de un cilindro de diámetro 10 cm. y altura 12 cm., es A) 1200 B) 300 C) 240 D) 120 E) 120 87. Unas pelotas se venden en latas de forma cilíndrica que contienen 3 pelotas cada una. Si el diámetro de la lata es de 6 cm. Calcular el volumen que queda sin ocupar en el interior de una lata. A) 18 cm3 B) 36 cm3 C) 54 cm3 D) 108 cm3 E) Ninguna de las anteriores 88. Una cúpula mide 8 m de diámetro, ¿cuál es su superficie si es semiesférica? A) 32 B) 64 C) 3 128  D) 3 256  E) 3 64  89. Calcular el volumen, en cm3 , de una pirámide cuadrada de 6 cm de lado y altura de una cara cm. A) B) C) 96 D) 288 E) Ninguna de las anteriores 90. Determinar el volumen, en cm3 , de una superficie esférica de 6 cm. de diámetro. A) 9 B) 12 C) 24 D) 36 E) 288 91. Calcular el volumen, en m3 , de un depósito cilíndrico de radio 3 m. y altura 4 m. terminado en una semiesfera. A) 54 B) 36 C) 30 D) 18 E) Ninguna de las anteriores
  • 14. Danny Perich C. www.sectormatematica.cl 92. Calcular el volumen generado por un triángulo equilátero de 2 cm. de altura al girar alrededor de ésta. A) 4 3 cm3 B) 64 3 3 cm3 C) 8  cm3 D) 16  cm3 E) 9 8 cm3 93. La diagonal de una de las caras de un cubo es 2 3 m. Calcular la superficie del cubo. A) 9 cm2 B) 18 cm2 C) 2 18 cm2 D) 27 cm2 E) 54 cm2 94. El área de una esfera es 48 cm2 . Aproximando el valor de  a 3, determinar la medida de su diámetro. A) 2 cm. B) 4 cm. C) 8 cm. D) 16 cm. E) Ninguna de las anteriores 95. A Mario lo contratan para pintar un recipiente cilíndrico de 20 m de diámetro y 15 m de altura, por el que cobra 750 pesos el metro cuadrado, ¿cuánto se le debe cancelar a Mario por el trabajo hecho, aproximando el valor de  a 3? A) $ 225.000 B) $ 675.000 C) $ 1.125.000 D) $ 1.350.000 E) $ 3.375.000 96. Si el largo de un paralelepípedo aumenta en un 25%, el ancho disminuye en un 20% y el alto se mantiene constante, entonces el volumen resultante, respecto del volumen original A) aumenta en 5% B) disminuye en 5% C) aumenta en 10% D) se mantiene constante E) disminuye 10% 97. El perímetro basal de una pirámide recta de base cuadrada es 10m, si la altura de la pirámide es 3m; entonces su volumen es A) 10 m3 B) 3 m3 C) 6,25 m3 D) 27 m3 E) 9 m3 98. Si se tiene una esfera de volumen V cm.3 y área de A cm.2 , el radio de dicha esfera en función de A y V es A) A V cm. B) V A3 cm. C) V A 3 3 cm. D) 3 2 A V 3 cm. E) A V 3 cm. 99. La capacidad de un cubo es 8 litros. La suma de todas las aristas del cubo es A) 160 cm. B) 200 cm. C) 220 cm. D) 240 cm. E) Otro valor 100. ¿Cuánto vale el volumen de un cono, si su altura es 2 3 y el perímetro de la base es 4 cm? A) 2 3 B) 6 2 C) 3 3 D) 2 4 E) Otro valor 101. Dos cuadriláteros A y B son semejantes. Los lados del cuadrilátero A son 10, 15, 18 y 12 cm. Si la constante de proporcionalidad es 3, ¿cuánto mide el menor de los lados de B? A) 3 , 0 cm. B) 3 , 3 cm. C) 5 cm. D) 6 cm. E) 30 cm.
  • 15. Danny Perich C. www.sectormatematica.cl 102. Los perímetros de dos polígonos semejantes P y Q son 45 y 54. El lado mayor de P es 15, ¿cuál es el lado mayor de Q? A) 5 B) 6 C) 15 D) 18 E) 24 103. En un triángulo las medidas de los ángulos interiores están en la razón 4 : 9 : 5. El triángulo es I) Isósceles II) Rectángulo III) Acutángulo A) Sólo I B) Sólo II C) Sólo III D) Sólo I y II E) Sólo I y III 104. Calcular la altura de un árbol que proyecta una sombra de 4,2 metros, si se sabe que un poste de 2,5 metros de altura proyecta, en el mismo momento, una sombra de 1,4 metros. A) 2,35 m. B) 4,2 m. C) 5,3 m. D) 7,5 m. E) 15 m. 105. Los triángulos ABC y DEF son semejantes. = 6 cm., = 12 cm., = 10 cm. y = 7,5 cm. Determinar + . A) 7,2 cm. B) 12,5 cm. C) 19,5 cm. D) 19,7 cm. E) 24,5 cm. 106. Tres árboles se encuentran alineados. El más pequeño mide 2 metros, el mediano mide 3,5 metros. Si la distancia entre cada árbol es de 15 metros, ¿cuánto mide el árbol más alto? A) 3,5 m. B) 5 m. C) 5,5 m. D) 7 m. E) 15 m. 107. Los lados de un polígono miden 6, 9, 12 y 15 cm. ¿Cuál es el perímetro del polígono semejante al anterior si su lado mayor mide 20 cm? A) 42 cm. B) 47 cm. C) 56 cm. D) 62 cm. E) Ninguna de las anteriores 108. La sombra de un edificio es de 50 metros y a esa misma hora la sombra de una casa de 5 metros de altura, es de 10 metros. ¿Cuál es la altura del edificio? A) 10 m. B) 25 m. C) 45 m. D) 50 m. E) 100 m. 109. En un triángulo isósceles las medidas del ángulo de la base y del vértice están en la razón 1:3; el ángulo mayor mide A) 36º B) 45º C) 90º D) 108º E) 135º 110. En un triángulo rectángulo los segmentos que la altura determina sobre la hipotenusa miden 16 y 36. El área del triángulo es A) 39 B) 78 C) 108 D) 216 E) 624 F E D C A B
  • 16. Danny Perich C. www.sectormatematica.cl 111. En la figura // . Determinar la medida de si = 20 cm, = 6 cm. y = 18 cm. A) 9 cm. B) 11 cm. C) 12,6 cm. D) 54 cm. E) 58 cm. 112. // ; = 15 cm., = 5 cm., = 3 cm., = A) 13 cm. B) 10 cm. C) 9 cm. D) 6 cm. E) 1 cm. 113. En la figura, // , entonces I) CD AC AB DE  II) EC BC DE AB  III) CD DE AC AB  A) Sólo I B) Sólo II C) Sólo III D) Sólo II y III E) I, II y III 114. En la figura, // , si = x + 3, = x + 2, = x + 8, = x + 6. La expresión que permite determinar x es A) 6 x 8 x 2 x 3 x      B) 3 x 8 x 6 x 2 x      C) 6 x 14 x 2 5 x 2 2 x      D) 14 x 2 8 x 3 x 5 x 2      E) 14 x 2 5 x 2    115. ABMN trapecio. = 8 cm, = 12 cm, = 15 cm. Entonces mide A) 22,5 cm. B) 11 cm. C) 10 cm. D) 6,4 cm. E) Ninguna de las anteriores 58° C D A B E 58° A D B E C A D B E C C N M B A S T R Q P
  • 17. Danny Perich C. www.sectormatematica.cl 116. ABCD es paralelogramo, = 12, = 4, = 18. Determinar . A) 54 B) 4 C) 6 D) 9 E) 12 117. En la figura, = a, = b, = c. Si // , entonces queda determinado por la expresión A) c ) b a ( a  B) b a ac  C) a bc D) b ac E) c ab 118. En la figura, // , entonces I) OP MO PQ MN  II) OQ ON OP OM  III) 2 OQ NO MN   IV) MN PQ PQ2   A) Sólo I B) Sólo II C) Sólo II y III D) Sólo II y IV E) Sólo I y II 119. ¿Cuál(es) de las siguientes relaciones se verifica(n) en la figura, siendo // y // ? I) CF AC BE AB  II) EF BC AE AB  III) AF AE AC AB  A) Sólo I B) Sólo II C) Sólo III D) Sólo I y II E) I, II y III D E C B A N Q M P O A B E C F D C F D E A B
  • 18. Danny Perich C. www.sectormatematica.cl 120. Si // , = 5, = 8, = 6, el valor de es A) 15,6 B) 9,6 C) 9 D) 6,6 E) 3,7 121. En la figura, ∆ABC  ∆DEF. Si  ; = = 5 cm. y = 4 cm. ¿Cuánto mide ? A) 2 cm. B) 3 cm. C) 4 cm. D) 5 cm. E) 6 cm. 122. Los triángulos ABC y DEF de la figura son congruentes, entonces la medida de es A) 8 B) 12 C) 15 D) 24 E) 30 123. En la figura siguiente, AC y BC son tangentes a la circunferencia de centro O. Si ACB = 70°, entonces el ABO = A) 20º B) 35º C) 45º D) 55º E) 70º 124. Desde un punto distante 5 cm. del centro de una circunferencia se ha trazado a ésta una tangente de 3 cm. de longitud. Determinar la medida del diámetro de la circunferencia. A) 2,5 cm. B) 4 cm. C) 5 cm. D) 8 cm. E) 10 cm. 125. En una circunferencia de centro O, se trazan los radios y , formando la cuerda , distinta al diámetro. Si AOB : BAO = 1 : 2. ¿Cuánto mide el ángulo inscrito en el arco AB? A) 18º B) 22,5º C) 36º D) 45º E) 72º O A C B O A C P M L D B A C B F D E 100 50 100 30 8 12 15 A B C D E F G
  • 19. Danny Perich C. www.sectormatematica.cl 126. En la circunferencia siguiente, = 3, = 12 y = 13. Calcular el menor valor del segmento . A) 1 B) 3,25 C) 4 D) 6,5 E) 9 127. Se tienen dos circunferencias concéntricas de radios 5 y 9 cm. En la circunferencia mayor, se traza la cuerda que intersecta a la menor en los puntos B y C de manera que = = . El valor de es A) 4 B) 8 C) 12 D) 16 E) Falta Información 128. = 2 cm., = 6 cm. y = 8 cm. El valor de en la siguiente circunferencia es: A) 40 B) 24 C) 4 D) 6 , 2 E) 1,5 129. Dos cuerdas se interceptan en una circunferencia formando trazos de medidas 2 y a + 8 en una de las cuerdas y medidas a + 3 y a + 2 en la otra. La cuerda mayor mide A) -5 B) 2 C) 9 D) 12 E) 20 130. Los radios de tres circunferencias tangentes exteriores miden 2, 4 y 6 cm, respectivamente. El área del triángulo que se forma al unir sus centros es A) 16 cm2 B) 18 cm2 C) 20 cm2 D) 24 cm2 E) No se puede determinar 131. En la figura, el segmento tangente mide 8 cm. y los segmentos exterior e interior, determinados por la secante miden 4 cm. y x cm., respectivamente. La medida de x es A) 2 B) 4 C) 12 D) 16 E) Ninguna de las anteriores 132. Una circunferencia está inscrita en el triángulo ABC, siendo P, Q y R los puntos de tangencia. ¿Cuánto mide el segmento de tangente CQ, sabiendo que el perímetro del triángulo es 40 cm, que = 7 cm. y = 15 cm? A) 5 B) 6 C) 8 D) 10 E) 12 A P D C B P A B C D
  • 20. Danny Perich C. www.sectormatematica.cl 133. En la figura siguiente = 3 m. y = 5 m., el valor de es A) 3 16 m. B) 3 4 m. C) 3 25 m. D) 2 5 m. E) 3 2 5  m. 134. Los catetos de un triángulo rectángulo miden 3 cm. y 4 cm. Determinar la proyección mayor de los catetos sobre la hipotenusa. A) 1,8 cm. B) 3,2 cm. C) 4 cm. D) 5 cm. E) 2 5 cm. 135. En la figura siguiente, = 6 cm.; = 3 cm. Determinar el área del triángulo ABC. A) 9 cm2 B) 12 cm2 C) 15 cm2 D) 18 cm2 E) 45 cm2 136. La altura hc de un triángulo ABC, rectángulo en C, es de 4 metros. Si los segmentos determinados sobre la hipotenusa están en la razón 1:2, ¿cuánto mide el área del triángulo ABC? A) 2 m2 B) 2 2 m2 C) 2 4 m2 D) 2 6 m2 E) 2 12 m2 137. En un triángulo ABC, rectángulo en C, se traza la altura . Si = 3,2 m. y = 5 m.; el valor de es A) 1,8 m. B) 3 m. C) 4 m. D) 76 , 5 m. E) 16 m. 138. En la figura, = 5-1 cm; = 2-1 cm; la altura del triángulo ABC es A) 10 1 B) 10 10 C) 10 D) 10 E) Ninguna de las anteriores 139. Los catetos de un triángulo rectángulo miden 3 cm. y 4 cm. Determinar la altura sobre la hipotenusa de este triángulo. A) 5 9 cm. B) 5 12 cm. C) 5 16 cm. D) 5 cm. E) Ninguna de las anteriores C D B A C D B A C D B A
  • 21. Danny Perich C. www.sectormatematica.cl 140. = 12 cm; = 9 cm; = A) 3 3 cm. B) 6 cm. C) 3 6 cm. D) 36 cm. E) Ninguna de las anteriores 141. Los catetos de un triángulo rectángulo están en la razón 3 : 4. Si la hipotenusa mide 10 cm., entonces el cateto menor mide A) 2 cm. B) 3 cm. C) 3,6 cm. D) 6 cm. E) 8 cm. 142. = 10 cm; = (p + 2) cm; = 2p cm; = ? A) 3,6 cm. B) 4 cm. C) 4,8 cm. D) 6,4 cm. E) 22,04 cm. 143. En la figura siguiente, AB // DE , 3 : 2 AD : CD  y 24 AB  , entonces la medida de segmento DE es igual a A) 8 B) 9,6 C) 12 D) 15 E) 16 144. En la figura, O es el centro de la circunferencia circunscrita al triángulo ABC y 3 OD  . ¿Cuánto mide el radio de la circunferencia circunscrita al triángulo ABC? A) 2 B) 3 C) 1 D) Falta información E) Otro valor 145. En la figura, AF = 40 cm. ha sido dividido en distintas partes. AB = 10 cm., AD = 2 AB – 3 cm. y DE = EF + 5 cm. ¿Cuánto mide EF ? A) 7 cm. B) 8 cm. C) 9 cm. D) 14 cm. E) 18 cm. A B C D O 60 o 3 A B D E F C D B A C D B A C D E A B
  • 22. Danny Perich C. www.sectormatematica.cl 146. En un triángulo cualquiera ABC se traza la bisectriz . Si = c, = b y = a. ¿Cuál es el valor de ? A) ac B) 2 ac C) b ac D) c ab E) c b a2 147. En la figura: ABC es equilátero y DCB es recto. ¿Cuál(es) de las siguientes afirmaciones es(son) verdadera(s)? I. 2· = + DAC es isósceles III. 2 = 2 + 2 A) Sólo I y II B) Sólo I y III C) Sólo II y III D) I, II y III E) Ninguna de ellas 148. ¿Cuál es la altura del trapecio isósceles si = 4 cm., = 10 cm. y = 12 cm? A) 4 cm. B) 3 4 cm. C) 6 cm. D) 3 6 cm. E) 8 cm. 149.  , ABC = 90º, = 6, = 4, = 3. ¿Cuál es el valor de en la siguiente figura? A) 9 B) 5 C) 9 5 D) 5 9 E) Falta información. 150. Determine la medida de una cuerda , sabiendo que O es centro de la circunferencia de radio 12 cm. y que la distancia del centro a la cuerda es de 9 cm. A) 3 3 cm. B) 7 3 cm. C) 7 6 cm. D) 15 cm. E) 30 cm. 151. Las circunferencias de centros P y Q son congruentes de radio 3 cm. cada una. ¿Cuánto mide PQ si OP 2 AB 3  A) 8 cm. B) 10 cm. C) 12 cm. D) 15 cm. E) 18 cm. D B C A A B D C A C B D E P Q A B
  • 23. Danny Perich C. www.sectormatematica.cl 152. En el círculo de la figura el diámetro es perpendicular a la cuerda . ¿Cuál es la medida de , si el radio de la circunferencia mide 12 cm. y = 8 cm? A) 12 cm. B) 8 cm. C) 2 8 cm. D) 2 4 cm. E) 4 cm. 153. En la figura // // . Si = 12 cm., = 8 cm., = 10 cm. ¿Cuál es el valor de ? A) 12 B) 13 C) 14 D) 15 E) Ninguna de las anteriores. 154. En un triángulo rectángulo, los trazos que la altura determina sobre la hipotenusa miden 8 y 18 cm. Entonces el área del triángulo es A) 78 cm2 B) 156 cm2 C) 312 cm2 D) 624 cm2 E) Ninguna de las anteriores 155. En el ABC de la figura, = , // ; // ; = 4 cm, = 2 cm; + = A) 3 B) 6 C) 4 D) Falta información E) Ninguna de las anteriores. 156. En el cuadrilátero ACBD, AB es bisectriz del  CAD y CD AB  . Si = = 6 cm. y = 4 cm. ¿Cuánto mide ? A) cm. B) 8 cm. C) 7 cm. D) 4 cm. E) Ninguna de las anteriores. 157. En la circunferencia de centro O y radio r, los triángulos MNO y MNT son isósceles congruentes. Entonces, = A) 2 3 r B) 3 r C) 3 r 2 D) 2 r E) 2 r 2 A D B C E F E D C B A C D E A F B A B C D E B O M T N
  • 24. Danny Perich C. www.sectormatematica.cl 158. En la figura, PT es tangente en T a la circunferencia de centro O y radio 6 cm. si = , el arco TQ mide A) 12cm. B) 9cm. C) 6cm. D) 3cm. E) 2cm. 159. En la figura se tiene que // ; = 3 ; =16; = A) 10 B) 12 C) 14 D) 16 E) 18 160. Dados dos lados de un triángulo miden 8 y 12 cm., entonces, el tercer lado puede medir I. 4 II. 8 III. 24 A) Sólo I B) Sólo II C) Sólo III D) Sólo I y II E) Sólo I y III 161. ABC y BDE son triángulos equiláteros congruentes de lado 8. Si =4 , ¿cuánto mide ? A) B) C) D) E) 162. En un triángulo ABC rectángulo en C cuya hipotenusa mide p, la medida de la proyección de un cateto sobre ella es m. ¿Cuál de las siguientes expresiones siempre representa al cuadrado de la medida del otro cateto? A) pm B) p2 - m2 C) (p - m)2 D) (pm)2 E) p2 - pm 163. Se tiene que = a; = 2a – 17; = a – 15 y = a + 2. Para que 1 L sea paralela a 2 L , el valor de AD debe ser I) 5 II) 7 III) 15 A) Sólo I B) Sólo II C) Sólo III D) Sólo I y II E) Sólo I y III P Q O T A B C D E A F B C D E A L1 L2 D E B C
  • 25. Danny Perich C. www.sectormatematica.cl 164. ¿Cuánto vale la base de un trapecio cuya mediana vale 3x y su otra base vale 2x? A) 4x B) x C) 6x D) 5x E) Otro valor 165. Desde un punto se traza un segmento tangente a una circunferencia de 16 cm. y una secante, cuyo segmento exterior mide 8 cm. El segmento interior de esta secante mide A) 24 cm. B) 31 cm. C) 96 cm. D) 192 cm. E) Ninguna de las anteriores 166. Un trazo AB está dividido interiormente en la razón 4 : 9, si el menor de los trazos mide 32 cm. ¿Cuál es la longitud del trazo? A) 32 B) 72 C) 104 D) 108 E) 120 167. L1 // L2. Si = 2x; = x + 2; = 4x – 1 y = 2x + 3; el valor de x es A) 2 B) 7 C) 4 D) -2 E) -1 168. Si en un triángulo rectángulo, p y q son las medidas de cada cateto respectivamente y r es la longitud de la hipotenusa, entonces siempre ocurre que A) p > r B) q > r C) r > p + q D) p < q E) q < r 169. Dado el triángulo ABC, de lados = 13; = 5 y = 12, entonces la altura desde el vértice C mide A) 13 30 B) 3 C) 4 D) 13 60 E) 5 170. La medida de los lados de un triángulo ABC son 6 cm., 8 cm. y 10 cm. El área del triángulo formado por los puntos medios de los lados respectivos es A) Un cuarto del área del triángulo ABC B) La tercera parte del triángulo ABC C) La mitad del triángulo ABC D) El doble del área del triángulo ABC E) No se puede determinar 171. La base de un triángulo y su altura están en razón 1 : 2. Si la base mide 16 cm. ¿Cuánto medirá el lado de un cuadrado de área equivalente? A) 8 cm. B) 16 cm. C) 64 cm. D) 32 cm. E) 24 cm. A B C A L1 L2 D E B C
  • 26. Danny Perich C. www.sectormatematica.cl 172. En un triángulo rectángulo, un cateto es el doble del otro y su perímetro es ) 5 3 ( 4  . ¿Cuál es su área? A) 32 cm2 B) 16 cm2 C) 5 4 cm2 D) ) 5 3 ( 4  cm2 E) Faltan datos 173. En el triángulo rectángulo de la figura se verifica que A) CD AB AC 2   B) BD AD BC 2   C) AB BC AC   D) BC AC CD 2   E) BD AB BC 2   174. Si = , = 16, = 4. Entonces el área de la circunferencia es A) 5 B) 10 C) 20 D) 25 E) 2,5 175. ¿Cuál(es) de las siguientes afirmaciones es(son) verdadera(s), de acuerdo a la figura siguiente? I)  =  II)  =  III)  =  A) Sólo I B) Sólo II C) Sólo III D) Sólo I y II E) Sólo II y III 176. El área de un rectángulo es 10 m2 ; el largo es el doble del ancho. La medida de una de sus diagonales es A) 10 m B) 5 m C) 25 m D) 10 m E) 4 m 177. En el círculo de la figura CD AB . ¿Cuál es la medida de CE, si el radio de la circunferencia mide 12 cm. y cm 8 BE  ? A) 12 cm. B) 8 cm. C) cm 2 8 D) cm 2 4 E) 4 cm. A E B D C A O C B E D O A B C D E C D B A
  • 27. Danny Perich C. www.sectormatematica.cl 178. La diagonal de un cuadrado mide 10 cm. Al disminuir su lado en 2 3 2 cm., la diagonal disminuye en A) 3 cm. B) 12 cm. C) 3 3 10 cm. D) 3 26 cm. E) 3 4 cm. 179. La suma de los ángulos interiores de un heptágono es A) 360º B) 1080º C) 900º D) 1260º E) 1440º 180. En la figura, la tangente mide 6 cm. y la secante = 18 cm, entonces mide A) 16 cm. B) 6 cm. C) 9 cm. D) 4 cm. E) 2 cm. 181. En la figura, se tiene que BCDE es un trapecio. Si = x + 4; = 2x – 2; = x y = x + 6; entonces, el valor de x es A) 2 B) 4 C) 6 D) 12 E) Otro valor 182. La medida de un ángulo exterior de un polígono regular de 18 lados es A) 10º B) 20º C) 30º D) 40º E) 50º 183. En el triángulo ABC de la figura, las transversales de gravedad CE y AD se interceptan en ángulo recto. Si GD =3 y GE=2, entonces BC mide A) 13 2 B) 17 2 C) 18 2 D) 10 E) 8 184. El perímetro de un hexágono inscrito en una circunferencia es 30 cm. Al calcular el área de la circunferencia resulta A) 30 2 cm  B) 36 2 cm  C) 10 2 cm  D) 5 2 cm  E) 25 2 cm  R P B A A E B D C G A L1 L2 D E B C
  • 28. Danny Perich C. www.sectormatematica.cl 185. En la circunferencia de centro O y radio r, MN es diámetro, si MP =r y Q punto medio de MP , entonces QN= A) 3 r B) 2 3 r C) 2 13 r D) 21 r E) No se puede determinar 186. En la figura, 1// 2. Si AC 4 = CE, entonces BC con DE , respectivamente, están en la razón A) 1 : 4 B) 1 : 5 C) 4 : 1 D) 5 : 1 E) ninguna de las anteriores 187. En la figura, ABC rectángulo en B, altura; además, // . Si AB = 2 5 cm. y AM = 5 cm., entonces DC = A) 5 cm. B) 2 5 cm. C) 10 cm. D) 2 10 cm. E) 2 25 cm. 188. Desde un punto situado a 40 cm. del centro de una circunferencia de 48 cm. de diámetro, se traza una tangente. ¿Cuál es su longitud? A) 8 B) 8 6 C) 4 D) 32 E) otro valor 189. Si dos polígonos son semejantes. Entonces ¿cuál(es) de las siguientes proposiciones es (son) verdadera(s)? I. Los polígonos tienen ángulos correspondientes de igual medida. II. Sus lados correspondientes son proporcionales. III. Los polígonos tienen la misma forma. A) sólo I B) sólo I y II C) sólo I y III D) sólo II y III E) Todas 190. En la figura se tiene = 3; = 3,5; = 4; = 1,5; el ∆AMN ~ ∆ABC. ¿Cuál es el perímetro del ∆ABC? A) 15,75 B) 13,25 C) 14,5 D) 14,55 E) Otro valor A B C D E L1 L2 M P N Q O A C B D M
  • 29. Danny Perich C. www.sectormatematica.cl 191. Un árbol da una sombra de 3 metros y, a la misma hora, un mástil de 4 metros de altura proyecta una sombra de 2 metros. ¿Cuál es la altura del árbol? A) 2 m. B) 3 m. C) 6 m. D) 12 m. E) otro valor 192. Si los triángulos de la figura son semejantes, entonces el perímetro y área del triangulo A’B’C’ son respectivamente A) 18 cm. y 13,5 cm2 B) 12 cm. y 6 cm2 C) 8 cm. y 4 cm2 D) 8 cm. y 3 8 cm2 E) 18 cm. y 9 cm2 193. ¿Cuál de las siguientes afirmaciones no es verdadera? A) Dos polígonos congruentes son siempre semejantes. B) Dos triángulos equiláteros son siempre semejantes. C) Dos cuadrados son siempre semejantes. D) Dos círculos son siempre semejantes. E) Dos rectángulos son siempre semejantes. 194. Los lados de un triangulo miden 5, 6 y 8 cm. ¿Cuánto miden los lados de un triangulo semejante si su lado más grande mide 16 cm? A) 6, 7 y 16 cm. B) 14, 15 y 16 cm. C) 10, 12 y 16 cm. D) 9, 12 y 16 cm. E) otras medidas. 195. Una fotografía de 14 cm. de largo por 10 cm. de ancho esta puesta en un marco que mide 2 cm. por lado más que la foto. ¿Cuál es la razón de semejanza entre el área de los rectángulos que forman el marco y la fotografía? A) 3 : 4 B) 5 : 4 C) 7 : 5 D) 9 : 5 E) otra razón 196. Una niña que mide 1 m proyecta una sombra de 2 m de largo. Si a esa misma hora y en ese mismo lugar, un árbol proyecta una sombra de 8 m de largo, ¿cuál es la altura del árbol? A) 4 m B) 5 m C) 6 m D) 7 m E) 16 m 197. Dos triángulos semejantes tienen áreas de 144 cm2 y 81 cm2 . La base del triángulo mayor mide 30 cm. ¿Cuánto mide la base triangulo menor? A) 22,5 cm. B) 16,875 cm. C) 25 cm. D) 24 cm. E) Otro valor. 198. Si ABC DEF, donde es homólogo con , = a cm y = 3a cm, ¿cuál de las siguientes afirmaciones es siempre verdadera? A) Si el área del triángulo ABC es 16 cm2 , entonces el área del triángulo DEF es 48 cm2 . B) 3∙ABC = DEF C) El perímetro del triángulo ABC es un tercio del perímetro del triángulo DEF. D) // , // y // E) Ninguna de las anteriores.
  • 30. Danny Perich C. www.sectormatematica.cl 199. Para que valor de x se tiene que 1 // 2? A) 6,5 B) 8 C) 6 D) 5 E) 4 200. Dado un triangulo ABC, se dibuja una recta paralela a que corta en D y a en E. Los segmentos y se cortan en X. ¿Cuál criterio de semejanza puede utilizar para probar de la forma mas simple que los triángulos DXE y BXC son semejantes? A) AA B) ALA C) LLA D) LAL E) No son semejantes. 201. En la figura 1 // 2, entonces se cumple A) d b c a  B) c b d a  C) d c b a  D) ad = bc E) b c d a  202. En la figura, si el ángulo en B es recto, ¿Cuál es la medida de x? A) 12,25 B) 11 C) 11,25 D) 11,75 E) 12,75 203. Si ∆ABC ~ ∆DEF, ¿Cuál(es) de las siguientes expresiones es (son) equivalente(s) a ? I) EF DE BC  II) BC AC EF  III) DF DE AC  A) sólo I B) sólo II C) sólo III D) sólo I y II E) sólo I y III 204. ¿A qué distancia del extremo P debe estar el punto A para que se cumpla : = 7 : 9; si = 36 cm? A) 18, 5 cm. B) 20 cm. C) 24 cm. D) 25 cm. E) 28 cm.
  • 31. Danny Perich C. www.sectormatematica.cl 205. ¿Cuánto vale x en la figura? A) 6,25 B) 16 C) 3,5 D) 7 E) 4 206. Los perímetros de dos figuras semejantes son 30 cm. y 18 cm. ¿En qué razón están los lados? A) 25 : 9 B) 10 : 9 C) 6 : 2 D) 5 : 3 E) 5 : 2 207. En la figura, ¿Cuál es el valor de x, sabiendo que los ángulos a y b son congruentes? A) 40 B) 22,5 C) 8 D) 8,4 E) 21 208. En la figura, = ; = 3; = 12; // . ¿Cuánto mide ? A) 15 B) 18 C) 16 D) 21 E) 20 209. En la figura 1 // 2 // 3 // 4, si = 60, entonces la medida es A) 12,75 B) 18,75 C) 11,25 D) 18,785 E) 30,75 210. En la figura, el área del triangulo ABC es 90 cm2 y // . ¿Cuál es el área del trapecio ABED? A) 36 cm2 B) 40 cm2 C) 50 cm2 D) 54 cm2 E) 60 cm2
  • 32. Danny Perich C. www.sectormatematica.cl 211. En los triángulos ABC y DEF de la figura, se sabe que // , // , = = 4, = = 8, y = 6, entonces el área del triangulo ABC es A) 180 B) 120 C) 108 D) 72 E) 54 212. Si =16; = 4; entonces = A) 8 B) 48 4 C) 3 4 D) 3 8 E) 2 8 213. Los lados de un triangulo están en la razón 2 : 3 : 5 y su perímetro mide 55 cm. ¿Cuánto mide el lado menor del triángulo? A) 12 cm. B) 11 cm. C) 10 cm. D) 9 cm. E) 7,5 cm. 214. En el triangulo ABC de la figura, es bisectriz del ángulo ACB. Entonces el perímetro de este triangulo es A) 57 cm. B) 51 cm. C) 60 cm. D) 15 cm. E) 81 cm. 215. En la figura, los puntos P, Q, R y S están sobre la circunferencia de centro O. Si : = 3 :4, = 6 , es el triple de , entonces mide A) 4 B) 6 C) 8 D) 9 E) 10 216. En la figura, el lado del ∆ABD es el diámetro de la circunferencia de centro O. Para el punto E en el lado , se tiene que = 3, = 12 y = 6. El valor del radio es A) 2 270 B) 270 C) 2 352 D) 2 352 E) 2 252
  • 33. Danny Perich C. www.sectormatematica.cl 217. En la circunferencia de diámetro = 15 cm., = 3 cm. y = 4 cm. ¿Cuál es la longitud de la cuerda ? A) 13 cm. B) 15 cm. C) 7 cm. D) 15,25 cm. E) 9 cm. 218. ¿Qué significa que dos triángulos sean semejantes? A) Que tienen igual área. B) Que tienen igual perímetro. C) Que sus lados son proporcionales. D) Que sus tres lados respectivos coinciden E) Que sus ángulos son proporcionales, en razón distinta de uno. 219. Según la figura ¿cuál(es) de las siguientes afirmaciones es (son) cierta(s)? I) ΔACD ~ ΔCBE II) ΔBEC ~ ΔAEB III) ΔACD ~ ΔCAB A) Sólo I B) Sólo II C) Sólo III D) Sólo I y III E) I, II y III 220. En la figura = 12, = 4, AT es tangente a la circunferencia de centro O, entonces mide A) 36 B) 32 C) 18 D) 16 E) 8 221. En la figura, si = 1 cm. y = 6 cm., entonces ¿cuánto mide ? A) 5 cm. B) 6 cm. C) 26 cm. D) 6 cm. E) 25 cm.
  • 34. Danny Perich C. www.sectormatematica.cl 222. En el ΔABC de la figura, se sabe que = 90 cm., = 12 cm. // // y : : = 1 : 2 : 3, entonces el valor de es A) 96 cm. B) 72 cm. C) 48 cm. D) 36 cm. E) 24 cm. 223. En la figura es un diámetro de la circunferencia de centro O y radio r. es tangente en P y mide r. Si M es el punto medio de , entonces la longitud de , en términos de r, es A) r B) 2 5 r C) 2 3 r D) 2 2 r E) 3 4r 224. C es el punto de tangencia de con la circunferencia. Para ambas figuras el cálculo correcto de x esta dado por A) DB AD x   2 B) 2 2 2 DB AD x   C) 2 2 2 BD BC x   D) BA AD x   2 E) 2 2 2 BD BC x   225. Si en la circunferencia de diámetro 30 cm. de la figura, la distancia desde el centro O de ella, hasta la cuerda es de 9 cm., entonces la cuerda mide A) 6 cm. B) 12 cm. C) 18 cm. D) 20 cm. E) 24 cm. 226. En la circunferencia de la figura se han trazado dos cuerdas con las medidas que se indican. La relación correcta es A) x = y B) x: y = 1: 3 C) y x 2 1  D) xy = 75 E) x + y = 20
  • 35. Danny Perich C. www.sectormatematica.cl 227. Desde un punto situado a 17 cm. del centro de una circunferencia de 8 cm. de radio se dibuja una tangente a la circunferencia, ¿Cuánto mide la tangente? A) 17 cm. B) 2 6 cm. C) 12 cm. D) 5 cm. E) ninguna de las anteriores. 228. La tarjeta de la figura está dividida en cuatro partes, cada una de ellas semejante a la tarjeta original. El valor de x es A) 5 B) 10 C) 15 D) 20 E) 30 229. En una circunferencia de centro O, el diámetro = 2 cm. R es un punto fuera de ella. S es la intersección de con la circunferencia y es tangente a ella. Si = , entonces mide A) 3 cm. B) 6 cm. C) 2 cm. D) 2 6 cm. E) 2 3 cm. 230. En la figura se tiene que es diámetro. La medida del arco CA es el doble de la del arco BC. Si mide 6 cm. ¿Cuántos centímetros cuadrados de área tiene el triángulo ABC? A) 9 B) 18 C) 3 9 D) 3 18 E) 36 231. En la figura 1 // 2, PQ = 6 cm.  L2. El área del triangulo PQS es 36 cm2 . mide A) 12 cm. B) 18 cm. C) 6 cm. D) 9 cm. E) no se puede determinar. 232. En la figura el triángulo ABC es rectángulo en A. = 10, = 6, = ,  , // . ¿Cuál es la medida de ? A) 3 20 B) 8 C) 15 D) 13 2 E) 6,5
  • 36. Danny Perich C. www.sectormatematica.cl 233. En la figura // // ; // ; = 8; = 18. Entonces = A) 13 B) 10 C) 11 D) 12 E) 15 234. En la figura ABCD es cuadrado de área 144 cm2 y EFCD es rectángulo de área 36 cm2 . ¿Cuál es el perímetro del triangulo AFE? A) 30 cm. B) 18 cm. C) 27 cm. D) 36 cm. E) 108 cm. 235. En el trapecio ABCD de la figura está formado por el triangulo equilátero AED y el rombo EBCD. Si la altura del triangulo es de 3 3 , ¿Cuál es el área del rombo? A) 9 cm2 B) 3 9 cm2 C) 13,5 cm2 D) 3 18 cm2 E) 81 cm2 236. En la figura ABC es triangulo rectángulo en C, // , // y = 10 cm. Si AB 5 4 BC  y : = 1 : 2. ¿Cuál es el área de la región achurada? A) 24 cm2 B) 8 cm2 C) 6 cm2 D) 12 cm2 E) 13 cm2 237. ¿Cuál es el área del la región achurada del triangulo rectángulo en C de la figura si es altura, = 29 cm. y = 4 cm? A) 6 cm2 B) 8 cm2 C) 16 cm2 D) 18 cm2 E) 20 cm2 238. En la figura = 10 cm., = 4 cm. ¿Cuántos centímetros cuadrados de área tiene el triángulo DBC? A) 72,5 B) 125 C) 20 D) 40 E) 80
  • 37. Danny Perich C. www.sectormatematica.cl 239. En el triangulo ABC, // . Si = x + 4; = x + 6; = x y = x + 1. ¿Cuál es el valor de x? A) 4 B) 3 C) 2 D) 1 E) otro valor 240. En el triangulo ABC, = 10 y = 4, ¿en qué razón están las áreas de los triángulos ADC y ABC? A) 2 : 3 B) 2 : 5 C) 3 : 7 D) 3 : 2 E) 3 : 5 241. En la circunferencia de centro O y radio 12 cm de la figura, = 5 cm. ¿Cuánto mide el segmento AC? A) cm B) cm C) 7 cm D) cm E) Indeterminable con los datos dados. 242. La circunferencia de centro O de la figura tiene diámetro 20 cm. Si = 4 cm. ¿cuál es el área del triangulo AOB? A) 20 cm2 B) 48 cm2 C) 40 cm2 D) 30 cm2 E) 96 cm2 243. En la figura O es centro de la semicircunferencia. Si = y  . ¿Cuál(es) de las siguientes proposiciones es (son) verdadera(s) considerando que = r? I. = r II. = 3 2 r III. CBD = 2CDB A) Sólo I B) Sólo III C) Sólo I y II D) Sólo I y III E) I, II y III
  • 38. Danny Perich C. www.sectormatematica.cl 244. En el triángulo ABC de la figura, // , = 45 cm., = 5 cm. y : = 4 : 5. ¿Cuál es el valor de ? A) 30 cm. B) 36 cm. C) 45 cm. D) 20 cm. E) 25 cm. 245. Si en la circunferencia de centro O de la figura, = , con y diámetros, entonces la medida del ángulo ACD es A) 45º B) 90º C) 15º D) 60º E) 30º 246. Desde un punto exterior a una circunferencia se traza una secante de 16 cm. que determina una cuerda de 5 cm. Si el punto está a 15 cm. del centro de la circunferencia, el radio de ella mide A) 5 cm. B) 6,5 cm. C) 7 cm. D) 7,5 cm. E) 8 cm. 247. En una circunferencia y son cuerdas. La intersección de ambas cuerdas es M. = 12, = 8, = 2. Entonces = A) 64 B) 32 C) 16 D) 4 E) 1 248. En la figura, el cuadrado tiene lado de medida a, además : = 1 : 4. Entonces la longitud de es A) B) C) D) E) 249. Los catetos de cierto triangulo rectángulo miden 12 y 5 centímetros. ¿Cuál es la medida de la altura correspondiente a la hipotenusa? A) B) C) D) E)
  • 39. Danny Perich C. www.sectormatematica.cl 250. En la figura, 1 y 2 son secantes, 3 es el lugar geométrico de los puntos que equidistan de 1 y 2. Si P es un punto de la recta 3, se puede afirmar que: I. Los triángulos ABP y CDP tienen la misma altura. II. La razón entre las aéreas de los triángulos ABP y CDP es igual a la razón entre la bases y . III. El BPC es ángulo recto siempre. A) solo I B) solo II C) solo III D) solo I y II E) Solo I y III 251. En el triangulo ABC se tiene que y son transversales de gravedad y se interceptan en forma perpendicular en G. Si = 3 y = 2, entonces mide: A) 2 B) 2 C) 2 D) 10 E) 18 252. En una circunferencia de 20 m de diámetro, la distancia desde el centro a una cuerda es 6 m. La cuerda mide: A) 8 m. B) 10 m. C) 12 m. D) 16 m. E) no es posible de calcular 253. En la figura, es un diámetro de la circunferencia de centro O y radio r. es tangente en P y mide r. Si M es el punto medio de , entonces la longitud de , en términos de r, es A) r B) C) D) E) 254. Se tiene un triangulo equilátero ABC de lado 4. D es un punto de y E es un punto de , además y son perpendiculares entre sí, lo mismo que y . Entonces mide A) 2 B) 3 C) D) 2 E) 255. Una circunferencia de centro O y radio 3 es tangente interiormente a una circunferencia de diámetro . Si A es el punto de tangencia, O está en , cuya medida es 8 cm. y es una cuerda de la circunferencia mayor, tangente a la circunferencia menor en C, ¿cuánto mide ? A) 1,8 cm. B) 2,4 cm. C) 2,5 cm. D) 3,0 cm. E) 3,6 cm.
  • 40. Danny Perich C. www.sectormatematica.cl 256. Se tienen dos circunferencias concéntricas, un segmento de medida 2 cm., el punto N perteneciente a la circunferencia exterior y el punto M a la circunferencia interior. Si es tangente a la circunferencia interior, ¿cuál es el área del anillo? A) 2cm2 B) 4cm2 C) 2 cm2 D) 4 cm2 E) no se puede determinar. 257. En una circunferencia, los segmentos de una de dos cuerdas que se interceptan miden 8 y 9 cm respectivamente. Sabiendo que uno mide el doble del otro, las medidas del los segmentos de la otra cuerda son A) 5 cm y 10 cm B) 7 cm y 10 cm C) 8 cm y 16 cm D) 6 cm y 12 cm E) 7 cm y 14 cm 258. Sea ABC un triangulo equilátero de lado a, M es un punto del arco BC en la circunferencia circunscrita, como indica la figura. Si b ¿cuánto mide ? A) a – b B) C) D) E) no se puede determinar 259. En una circunferencia , y son cuerdas, corta a en P y a en Q. =9, = 4, = 10, = 2 y = 6. Entonces mide A) 4 B) 6 C) 10 D) 12 E) 20 260. En la circunferencia de centro O y diámetro de la figura,  . Si = 8 cm y = 4 cm, ¿cuánto mide el perímetro de la circunferencia? A) 16cm B) 20cm C) 32cm D) 80cm E) 100cm 261. Si el radio de un cono se duplica, entonces su volumen A) se mantiene igual B) se duplica C) se reduce a la mitad D) se cuadruplica E) se reduce a la cuarta parte 262. El triangulo ABC es rectángulo en C. es transversal de gravedad sobre . ¿Cuánto mide el ángulo b? A) 20o B) 110o C) 55o D) 70o E) 32,5o
  • 41. Danny Perich C. www.sectormatematica.cl 263. El área de un hexágono regular es de 24 luego de aplicar una homotecia se obtiene un hexágono regular de área 6 . ¿Cuál es el factor de homotecia? A) 0,5 B) 0,25 C) 2 D) 4 E) 1,5 264. Los lados de un rectángulo están en la razón 3 : 2. Si el lado mayor mide 3 unidades más que el lado menor, ¿cuánto mide el área del rectángulo? A) 108 cm2 B) 96 cm2 C) 54 cm2 D) 40 cm2 E) 24 cm2 265. En el triangulo ABC se tiene que es la bisectriz de ángulo BAC. =12 cm.; = 2 cm.; = 14 cm., el valor del segmento s es A) 5,5 cm. B) 10,5 cm. C) 12 cm. D) 6 cm. E) ninguna de las anteriores. 266. En la figura, dadas las dimensiones del rectángulo ABCD, entonces la medida del lado en el rectángulo DBEF mide A) B) C) D) E) 1 267. Considere un paralelogramo ABCD. Siendo M el punto medio del lado y O el punto de intersección del trazo con la diagonal ; entonces se tiene que = A) B) C) D) E) 268. Se tiene un cuadrado AFED y un triangulo BAC rectángulo en A, tal que, D está entre B y A; F está entre A y C; E está entre B y C; = 1 y = 3. ¿Cuánto mide el lado del cuadrado? A) 0,70 B) 0,75 C) 0,80 D) 0,85 E) 0,90 269. Un punto Q divide en sección áurea a un trazo , con > . Si = 10 cm. y = x, entonces la ecuación para determinar x es A) x2 + 10x – 100 = 0 B) x2 - 10x + 100 = 0 C) x2 - 10x – 100 = 0 D) x2 + 10x + 100 = 0 E) x2 + x – 100 = 0
  • 42. Danny Perich C. www.sectormatematica.cl 270. Dada una circunferencia de centro O y radio a, con ⊥ y = b, el valor = x está dado por la ecuación A) x2 - 2ax – b2 = 0 B) x2 - 2ax + b2 = 0 C) x2 + 2ax – b2 = 0 D) x2 + 2ax + b2 = 0 E) x2 - ax + b2 = 0 271. En la figura = 6,5; = 12, O centro de la semi-circunferencia. El área del triangulo ABC es A) 20 B) 30 C) 40 D) 60 E) 120 272. // entonces la medida de es A) 15 B) 16 C) 25 D) 32 E) otro valor 273. La cuerda es diámetro y la simetral de la cuerda ; P es punto de ; EP se extiende hasta cortar la circunferencia en A; entonces, el triangulo EPM es semejante con el triangulo A) EFA B) EFC C) ABM D) ABP E) FMC 274. En la figura = 12; = 7; = 9, entonces mide A) 12 B) 10,5 C) 7,5 D) 4 E) 6 275. La razón entre la altura del triangulo ABC y la altura correspondiente del triangulo MNP es 7:10. Si los triángulos son semejantes, ¿cuál es la razón entre las aéreas de estos triángulos? A) 7 : 10 B) 10 : 7 C) 49 : 100 D) 343 : 1000 E) 1000 : 343
  • 43. Danny Perich C. www.sectormatematica.cl 276. Dos octógonos regulares tienen lados cuyas longitudes son 6 y 9. La razón de sus aéreas es A) 4 : 3 B) 2 : 3 C) 2 : D) : 9 E) 2 : 1 277. Se tiene un segmento de 95 cm. Al dividirlo interiormente por dos puntos dados P y Q tales que : : = 3 : 5 : 11, la diferencia entre el mayor y el menor de los segmentos que resultan de tal división es A) 15 cm. B) 25 cm. C) 40 cm. D) 55 cm. E) 60 cm. 278. Las diagonales de un rombo miden 30 y 40. ¿Cuánto mide el radio de la circunferencia inscrita en él? A) 12 B) 15 C) 18 D) 21 E) 25 279. En el triangulo ABC de la figura adjunta, // . Además = a, = 1,4a y el área del triangulo ABC es 98 cm. El área del triangulo EFC es A) 35 cm. B) 42 cm. C) 49 cm. D) 50 cm. E) 56 cm. 280. Si MNPQ es un rectángulo y ⊥ , entonces, de acuerdo a los datos de la figura, es equivalente a A) B) C) D) E) 281. ¿Cuál(es) de las siguientes afirmaciones es (son) verdadera(s)? I) El rombo tiene ejes de simetría y centro de simetría. II) El romboide tiene centro de simetría, pero no tiene ejes de simetría. III) El trapecio isósceles tiene eje de simetría, pero no tiene centro de simetría. A) Solo I B) Solo I y II C) Solo I y III D) I, II y III E) Ninguna de ellas 282. El perímetro de un rectángulo es de 46 cm. Si el largo disminuye en 3 cm. y el ancho aumenta en 2 cm., el área del rectángulo no cambia. En estas condiciones, la diferencia de las medidas originales entre el largo y el ancho es A) 15 cm. B) 12 cm. C) 8 cm. D) 7 cm. E) 5 cm.
  • 44. Danny Perich C. www.sectormatematica.cl 283. Los lados de un triángulo miden 2 , 6 y 8 , respectivamente. ¿Cuál es la longitud de la menor de las alturas de este triángulo? A) 2 B) 6 C) 8 D) 2 2 E) 2 6 284. es altura del triángulo rectángulo ABC. Si : = 1 : 2 y = 4, ¿cuál(es) de las siguientes relaciones es(son) verdadera(s)? I) 2 > II) < III) 2 > A) Sólo II B) Sólo I y II C) Sólo I y III D) Sólo II y III E) I, II y III 285. ABCD es un cuadrado de lado a, en que lo divide en un triángulo y en un trapecio cuyas áreas están en la razón 1 : 4. ¿Cuál es el valor de ? A) 4 1 a B) 5 2 a C) 3 1 a D) 4 3 a E) 5 3 a 286. En la figura se tiene un círculo de centro O y otros tres semicírculos de radio 6 y centros O, O1 y O2 . ¿Cuál es el área de la región achurada? A) 3 – 3 B) 5 + 3 C) 7 – 3 3 D) 4 + 3 3 E) 8 – 3 287. La circunferencia más grande de la figura tiene radio R y las otras dos tienen radio r. Si 10   r R y E es centro de la circunferencia, entonces + = A) 95 B) 96 C) 97 D) 98 E) 100 288. La suma de dos ángulos exteriores de un triángulo es igual a 270°. Si el lado mayor mide 48 cm., ¿cuánto mide el trazo que une el circuncentro con el centro de gravedad? A) 6 cm. B) 8 cm. C) 12 cm. D) 16 cm. E) 20 cm.
  • 45. Danny Perich C. www.sectormatematica.cl 289. En la figura, ΔDFE es rectángulo en F, ADCB es trapecio isósceles de lados = = 5, = 4 y = 10. Si B es punto medio de , entonces = A) 3,25 B) 3,5 C) 3,75 D) 4,0 E) 4,25 290. Si en un rombo de área k, la diagonal mayor es el doble de la menor, entonces el lado del rombo en términos de k es A) k B) k 2 2 1 C) k 3 3 1 D) k 4 4 1 E) k 5 2 1 291. En la figura, P divide a en la razón 2 : 3 y Q divide a en la razón 3 : 4. Si = 2 cm, entonces la longitud de es A) 60 cm. B) 70 cm. C) 75 cm. D) 80 cm. E) 85 cm. 292. En la figura, se han dibujado 3 semicírculos, tangentes entre sí, con diámetros , y y. Si  , entonces la razón entre el área de la región achurada y el área del círculo de diámetro es A) 1 : 2 B) 1 : 3 C) 3 : 7 D) 1 : 1 E) 2 : 6 293. Un círculo de radio r está contenido en otro de radio R. Si el área del círculo mayor es b a veces el área de la región comprendida entre el círculo menor y el mayor, entonces r R es igual a A) b a B) b a a  C) b a b  D) b a a  E) b a b  294. En el triángulo ABC de la figura, se traza de manera tal que : = 1 : 2, dimidia a , luego : = A) 1 : 4 B) 1 : 3 C) 2 : 5 D) 4 : 11 E) 3 : 8
  • 46. Danny Perich C. www.sectormatematica.cl 295. En la figura 6, las líneas segmentadas son bisectrices de los ángulos interiores del rectángulo ABCD y la región achurada es un cuadrado. Si los lados del rectángulo son a y b, entonces para determinar el área del cuadrado se debe conocer (1) a – b (2) ab A) (1) por sí sola B) (2) por sí sola C) Ambas juntas, (1) y (2) D) Cada una por sí sola, (1) ó (2) E) Se requiere información adicional 296. La figura muestra un cuadrado de lado L en el cual se ha inscrito un rectángulo de lados a y b. Entonces, L en función de a y b, es igual a 2 b a ) A  B) 2 2 b a  C) ab D) 2 b a  E) ab 2 297. Si en un triángulo isósceles de base 2 cm, el ángulo del vértice mide 45º, entonces la medida de uno de sus lados iguales es A) 3 cm. B) 2 2 cm. C) ) 2 2 4 (  cm. D) 2 2 4  cm. E) Falta información 298. En la figura, // , = = y ABC = 34°, ¿cuánto mide el AEC? A) 34° B) 51° C) 67° D) 68° E) 90° 299. En el rectángulo ABCD de la figura, P es un punto interior tal que = 3 cm, = 4 cm y = 5 cm, ¿cuánto mide ? A) 3 2 cm. B) 2 3 cm. C) 3 3 cm. D) 2 4 cm. E) 2 cm.
  • 47. Danny Perich C. www.sectormatematica.cl 300. En el lado del triángulo ABC se tiene un punto E de forma que : = 1 : 3 y en el lado un punto D tal que : = 1 : 2. Si F es el punto de intersección de y , entonces FD AF FC EF  es igual a A) 5 4 B) 4 5 C) 2 3 D) 2 E) 2 5 301. ABCD es un cuadrado de lados 1 cm. En los lados y se toman, respectivamente, los puntos E y F, de modo que = . ¿Cuál es el área máxima del cuadrilátero CDFE? A) 2 1 cm2 B) 16 9 cm2 C) 32 19 cm2 D) 8 5 cm2 E) 3 2 cm2 302. En la figura, el triángulo PQR es equilátero y está formado por cuatro triángulos equiláteros. Si se hace girar la figura en torno a la altura del ΔPQR, entonces el volumen del sólido generado por el triángulo achurado de lado L es A) B) C) D) E) 303. Una esfera de volumen V se calienta hasta que su radio se incrementa en un 10%. ¿Cuál es el nuevo volumen de la esfera? A) 10 -3 V B) 1,1 V C) 1,21 V D) 1,030 V E) 1,331 V 304. En la figura, el área del triángulo OAD es igual a del área del trapecio isósceles OCBA. ¿Cuáles son las coordenadas del punto medio de ? A) (5, 5) B) (4, 5) C) (5, 6) D) (4, 3) E) (5, 4)
  • 48. Danny Perich C. www.sectormatematica.cl 305. En cierto triángulo ABC se dibuja la transversal de gravedad . En el triángulo DBC se dibuja la transversal de gravedad , que mide 9 cm. Sobre se toma un punto F de modo que // . ¿Cuánto mide ? A) 3 cm B) 4,5 cm C) 5 cm D) 6 cm E) 6,2 cm 306. En la figura, ΔABC es isósceles de base = b y altura = h. ¿Cuál es el área del rectángulo DEHG si = x? A) B) C) D) x(b – x) E) x(h – x) 307. El área de la figura es de 720 mm 2 . Si la figura está formada por un cuadrado, por dos trapecios isósceles congruentes y dos triángulos congruentes, todos de igual área, entonces ¿cuánto mide la distancia x? A) 16 mm B) 18 mm C) 20 mm D) 22 mm E) 24 mm 308. Se tiene el triángulo ABC isósceles rectángulo en A. Sus catetos miden 1. Si DF y DE , AD son radios de la semicircunferencia y DF es perpendicular a BC . ¿Cuánto vale el radio de la semicircunferencia inscrita? A) B) C) D) E) 309. El triangulo ABC de la figura es isósceles de base BC, si BD = BE y CB = CD, entonces ¿cuál(es) de la(s) siguientes afirmaciones es(son) verdadera(s)? I) BAC = DCB = EBD II) ∆ADC  ∆ CEB III) ∆BEC es isósceles A) Sólo I B) Sólo II C) Sólo I y II D) Sólo I y III E) Sólo II y III
  • 49. Danny Perich C. www.sectormatematica.cl 310. La hipotenusa de un triángulo rectángulo mide 10 cm y el radio de la circunferencia inscrita 1 cm. El perímetro del triángulo es A) 15 cm B) 22 cm C) 24 cm D) 26 cm E) 30 cm 311. La altura que llega a la base de un triángulo isósceles de perímetro 32, mide 8, el área del triángulo es A) 56 B) 48 C) 40 D) 32 E) 24 312. En la figura, ABCD es cuadrado, M es punto medio de y N es punto medio de . El cociente entre el área de AOCD y el área de ABCD es A) B) C) D) E) 313. El volumen engendrado al rotar un rombo de diagonales 6 y 8, por uno de sus lados, es A) B) C) D) E) Otro valor 314. En la figura, polígono de vértices A, B y C, cuyas coordenadas son: (-1, -2); (2, -2) y (2, 2), respectivamente. Si se le aplica una rotación de 90° con centro en A, ¿cuál será la coordenada del vértice C del polígono en la nueva posición? A) (1, -5) B) (-5, 1) C) (3, -5) D) (-5, 3) E) (3, 5) 315. Las coordenadas del punto (x, y), perteneciente al segundo cuadrante, después de una simetría central con respecto al origen del sistema cartesiano está representado por A) (x, y) B) (x, -y) C) (-x, y) D) (-x, -y) E) x y , 2 2       316. En la circunferencia de centro O de la figura, ¿cuál(es) de las siguientes afirmaciones es (son) verdadera(s)? I) ∙ = ∙ II) = ∙ III) ∙ = ∙ A) Solo I B) Solo II C) Solo I y III D) Solo II y III E) I, II y III
  • 50. Danny Perich C. www.sectormatematica.cl 317. Se tiene un triángulo ABC equilátero de altura . Se realiza una homotecia con centro C y razón = -2, el área del triángulo homotético A’B’C’ es A) 9 B) 12 C) 36 D) 42 E) 45 318. En la figura, se tienen dos circunferencias congruentes y tangentes exteriores de radio 6 cm. Si AB contiene los centros de las circunferencias y BT es tangente en T, entonces la medida de la cuerda BP es A) 12 2 B) 8 2 C) 6 2 D) 4 2 E) 3 2 319. En la circunferencia de la figura Nº 3, M es el punto medio del arco BMC, ⊥ AB. Si = x y = x + 1, entonces = A) 3x + 2 B) 3x + 1 C) 2x + 3 D) 2x + 2 E) 2x + 1 320. En la figura, ABCDEF es hexágono regular, O es el centro del hexágono, es perpendicular a y M es la intersección de y . Si = 8, entonces ¿cuánto mide ? A) 3 B) C) D) 4 E) A T P B
  • 51. Danny Perich C. www.sectormatematica.cl ALTERNATIVAS CORRECTAS 1 B 2 D 3 A 4 B 5 B 6 C 7 E 8 A 9 D 10 B 11 E 12 D 13 A 14 A 15 D 16 D 17 A 18 B 19 A 20 B 21 E 22 A 23 D 24 C 25 D 26 D 27 A 28 A 29 D 30 A 31 B 32 D 33 B 34 B 35 B 36 C 37 E 38 C 39 E 40 D 41 B 42 A 43 E 44 A 45 E 46 B 47 B 48 A 49 B 50 D 51 A 52 A 53 D 54 C 55 C 56 E 57 C 58 A 59 D 60 E 61 D 62 B 63 E 64 C 65 D 66 E 67 A 68 C 69 E 70 E 71 D 72 D 73 C 74 B 75 B 76 D 77 E 78 B 79 D 80 B 81 E 82 C 83 C 84 E 85 D 86 B 87 C 88 A 89 C 90 D 91 A 92 E 93 E 94 B 95 C 96 D 97 C 98 E 99 D 100 D 101 B 102 D 103 B 104 D 105 E 106 B 107 C 108 B 109 D 110 E 111 C 112 D 113 D 114 A 115 A 116 D 117 B 118 E 119 E 120 A 121 E 122 A 123 B 124 D 125 A 126 C 127 C 128 A 129 D 130 D 131 C 132 A 133 A 134 B 135 E 136 E 137 C 138 B 139 B 140 B 141 D 142 C 143 B 144 A 145 C 146 C 147 A 148 C 149 D 150 C 151 E 152 C 153 D 154 B 155 C 156 A 157 B 158 E 159 B 160 B 161 B 162 E 163 A 164 A 165 A 166 C 167 A 168 E 169 D 170 A 171 B 172 B 173 E 174 B 175 B 176 B 177 C 178 E 179 C 180 A 181 C 182 B 183 D 184 E 185 C 186 B 187 C 188 D 189 E 190 A 191 C 192 A 193 E 194 C 195 D 196 A 197 A 198 C 199 C 200 A 201 B 202 C 203 E 204 E 205 E 206 D 207 E 208 B 209 B 210 C 211 C 212 D 213 B 214 E 215 A 216 E 217 A 218 C 219 E 220 D 221 A 222 B 223 B 224 A 225 E 226 D 227 E 228 D 229 C 230 E 231 A 232 A 233 A 234 D 235 B 236 C 237 E 238 B 239 A 240 E 241 A 242 B 243 E 244 D 245 E 246 C 247 C 248 A 249 D 250 D 251 D 252 D 253 B 254 B 255 B 256 B 257 D 258 C 259 A 260 B 261 D 262 E 263 A 264 C 265 B 266 D 267 D 268 B 269 A 270 C 271 B 272 C 273 A 274 E 275 C 276 A 277 C 278 A 279 D 280 D 281 B 282 D 283 E 284 E 285 B 286 C 287 E 288 B 289 D 290 E 291 B 292 D 293 B 294 B 295 A 296 A 297 D 298 B 299 B 300 C 301 D 302 D 303 E 304 E 305 D 306 C 307 C 308 C 309 C 310 B 311 B 312 C 313 B 314 B 315 D 316 A 317 C 318 B 319 E 320 C